Basics of Sub-Math by Khalid M

You might also like

Download as pdf or txt
Download as pdf or txt
You are on page 1of 64

Khalid

Muhamood

BASICS OF
SUBSIDIARY
MATHEMATICS
Based on the 2020 NCDC syllabus

Khalid
MUHAMOOD 2023
EDITION
Basics of Subsidiary Mathematics

By

KHALID MUHAMOOD
MSc. Mathematical Science (AIMS - Rwanda)
BSc. Ed (HONS) (KYU - Uganda)

CONTACTS
TEL: +256 754857297
Email: khalidhkmsh@gmail.com

i
ACKNOWLEDGEMENTS
I extend my great appreciation to the Director and entire Staff of Hope Boarding Secondary School - Lutembe for
the support given towards the production of this book.

I would like to extend my sincere thanks to Mr. Ssegawa yasini of Kyambogo University Mathematics department,
Mr. Matama Peter, Head of Mathematics Department Kajjansi Progressive SS, Mr. Walugada Ronald of Ndejje
SSS. for their consistence guidance, encouragement, advice and in depth insight that resulted into the successful and
comprehensive compilation of this book.

ii
DEDICATION
This book is dedicated to all Subsidiary Mathematics Students in Uganda.

iii
INTRODUCTION
The objective of this book is to empower students with the techniques and knowledge essential for understanding and
excelling in Subsidiary Mathematics.

We recognize that both students and teachers play pivotal roles in the educational journey. Therefore, this book is
designed to benefit both. Students will find it an indispensable companion in their exam preparations, while teachers
will appreciate the resource it offers to aid their teaching endeavors.

As we embark on this mathematical journey together, we welcome your feedback and suggestions for the improvement
of this text. Our commitment to enhancing the learning experience knows no bounds, and your insights are invaluable.
Together, we can continue to refine and elevate the standards of mathematical education.

We wish you every success in your Subsidiary Mathematics examinations and trust that this guide will be a guiding
light on your path to excellence.

iv
Contents

Acknowledgements ii

Dedication iii

Introduction iv

1 ALGEBRA 1

1.1 MATRICES . . . . . . . . . . . . . . . . . . . . . . . . . . . . . . . . . . . . . . . . . . . . . . . . 1

1.2 INDICES, SURDS AND LOGARITHMS . . . . . . . . . . . . . . . . . . . . . . . . . . . . . . . . . 3

1.3 SEQUENCE, SERIES, SIGMA NOTATION . . . . . . . . . . . . . . . . . . . . . . . . . . . . . . . 6

1.4 Arithmetic Progressions APs and Geometric Progressions GPs . . . . . . . . . . . . . . . . . . . . . 7

1.5 FACTORIAL NOTATION, PERMUTATION AND COMBINATIONS . . . . . . . . . . . . . . . . . 11

1.6 Examples . . . . . . . . . . . . . . . . . . . . . . . . . . . . . . . . . . . . . . . . . . . . . . . . . 11

1.7 Exercises . . . . . . . . . . . . . . . . . . . . . . . . . . . . . . . . . . . . . . . . . . . . . . . . . 11

1.8 Permutations . . . . . . . . . . . . . . . . . . . . . . . . . . . . . . . . . . . . . . . . . . . . . . . 11

2 LINEAR PROGRAMMING 16

2.1 Introduction . . . . . . . . . . . . . . . . . . . . . . . . . . . . . . . . . . . . . . . . . . . . . . . 16

2.2 Inequalities and Graphs . . . . . . . . . . . . . . . . . . . . . . . . . . . . . . . . . . . . . . . . . 17

2.3 Optimization . . . . . . . . . . . . . . . . . . . . . . . . . . . . . . . . . . . . . . . . . . . . . . . 18

2.4 Exercise: . . . . . . . . . . . . . . . . . . . . . . . . . . . . . . . . . . . . . . . . . . . . . . . . . 20

3 MOVING AVERAGES, PRICE INDICES, CORRELATION 21

3.1 Moving Averages . . . . . . . . . . . . . . . . . . . . . . . . . . . . . . . . . . . . . . . . . . . . . 21

3.2 INDEX NUMBERS . . . . . . . . . . . . . . . . . . . . . . . . . . . . . . . . . . . . . . . . . . . . 32

3.3 REGRESSION AND CORRELATION . . . . . . . . . . . . . . . . . . . . . . . . . . . . . . . . . . 34

3.4 Scatter Diagram/Scatter Graph . . . . . . . . . . . . . . . . . . . . . . . . . . . . . . . . . . . . . 35

4 VECTORS AND TRIGONOMETRY 37

4.1 Vectors . . . . . . . . . . . . . . . . . . . . . . . . . . . . . . . . . . . . . . . . . . . . . . . . . . 37

4.2 Vector Operations . . . . . . . . . . . . . . . . . . . . . . . . . . . . . . . . . . . . . . . . . . . . 37

v
CONTENTS CONTENTS

4.3 Examples . . . . . . . . . . . . . . . . . . . . . . . . . . . . . . . . . . . . . . . . . . . . . . . . . 38

4.4 Trial Exercise . . . . . . . . . . . . . . . . . . . . . . . . . . . . . . . . . . . . . . . . . . . . . . . 39

4.5 Trigonometry . . . . . . . . . . . . . . . . . . . . . . . . . . . . . . . . . . . . . . . . . . . . . . . 39

5 DIFFERENTIATION, INTEGRATION AND CURVE SKETCHING 41

5.1 Derivatives . . . . . . . . . . . . . . . . . . . . . . . . . . . . . . . . . . . . . . . . . . . . . . . . 41

5.2 Critical Points and the First Derivative . . . . . . . . . . . . . . . . . . . . . . . . . . . . . . . . . 43

5.3 Formulation of Functions . . . . . . . . . . . . . . . . . . . . . . . . . . . . . . . . . . . . . . . . 43

5.4 Sketching Quadratic Curves . . . . . . . . . . . . . . . . . . . . . . . . . . . . . . . . . . . . . . . 45

5.5 Determining the Nature of Turning Points . . . . . . . . . . . . . . . . . . . . . . . . . . . . . . . 45

5.6 Intercepts . . . . . . . . . . . . . . . . . . . . . . . . . . . . . . . . . . . . . . . . . . . . . . . . . 46

5.7 Curve Sketching . . . . . . . . . . . . . . . . . . . . . . . . . . . . . . . . . . . . . . . . . . . . . 46

5.8 Integration . . . . . . . . . . . . . . . . . . . . . . . . . . . . . . . . . . . . . . . . . . . . . . . . 46

5.9 Indefinite Integrals . . . . . . . . . . . . . . . . . . . . . . . . . . . . . . . . . . . . . . . . . . . . 46

5.10 Definite Integrals . . . . . . . . . . . . . . . . . . . . . . . . . . . . . . . . . . . . . . . . . . . . . 47

5.11 Natural Logarithm in Integration . . . . . . . . . . . . . . . . . . . . . . . . . . . . . . . . . . . . 48

6 Probability Theory 49

vi
1. ALGEBRA
1.1 MATRICES
A matrix is a rectangular arrangement of numbers orga- Example (Column Matrix):
nized in rows and columns.  
3
Example: " # C = −2
 
2 4 1
A= 0
0 5 3

1.1.1 Matrix Notation. A matrix with m rows and n 1.1.5 Rectangular Matrix. A matrix with different
columns is denoted as m × n matrix. numbers of rows and columns is called a rectangular ma-
trix.
Example:  
1 −3 Example: " #
B = 2 7 
  1 2 3
P =
0 9 −1 0 1
Matrix P is a rectangular matrix of order 2 × 3.
1.1.2 Order of a Matrix. The order of a matrix is de-
termined by its dimensions: the number of rows by the 1.1.6 Addition and Subtraction. Matrices of the same
number of columns. dimensions can be added/subtracted element-wise.

Example: Example:
" #
2 3 −1 " # " #
M= 2 1 −1 2
0 1 4 C= D=
0 3 4 0
The order of matrix M is 2 × 3.
" #
1.1.3 Square Matrix. A square matrix has an equal 1 3
C +D =
number of rows and columns. 4 3

Example: #" 1.1.7 Scalar Multiplication. Each element of a matrix


2 −2 can be multiplied by a constant (scalar).
N=
1 3
Example:
Matrix N is a square matrix of order 2 × 2.
" #
3 −2
E=
1.1.4 Row and Column Matrices. A matrix with only 0 1
one row is called a row matrix, and a matrix with only " #
6 −4
one column is called a column matrix. 2E =
0 2
Example (Row Matrix):
1.1.8 Matrix-Vector Multiplication. Multiplying a
h i
R = 1 2 −1 matrix by a vector transforms the vector.

Basics of Subsidiary Mathematics By Khalid M +256 754857296 khalidhkmsh@gmail.com 1


1.1. MATRICES 1. ALGEBRA

Example: Example: " #


2 1
"
# # " K=
2 −1 4 1 3
F = v=
3 0 −2 " #
3/5 −1/5
K −1 =
−1/5 2/5
" #
10
Fv =
12
1.1.12 Determinant, Inverse, and Singular Matri-
1.1.9 Matrix-Matrix Multiplication. For matrix- ces. The determinant of a square matrix A can be calcu-
matrix multiplication, the column count of the first matrix lated using various methods, such as cofactor expansion
must match the row count of the second matrix. or row reduction.

Example: Example: Consider the matrix


" # " # " #
2 1 0 5 3 2
G= H= A=
−1 3 2 −2 1 4

"
# The determinant of A is given by:
2 8
GH =
4 −16
det(A) = 3 · 4 − 2 · 1 = 10
1.1.10 Non-Commutativity of Matrices. Matrix mul-
tiplication is not commutative, meaning the order mat- To find the inverse of a matrix A, you can use the for-
ters. mula:
1
A−1 = · adj(A)
Example: det(A)
" # " # where adj(A) is the adjoint of matrix A.
1 0 2 3
I= J=
0 1 4 5 Example: For matrix A,
" #
4 −2
" #
2 3 adj(A) =
IJ = −1 3
4 5
" #
14 19 Thus, the inverse of A is:
JI =
20 27 " #
1 4 −2
A−1 =
1.1.11 Identity Matrix and Inverse. The identity ma- 10 −1 3
trix is the matrix equivalent of the number 1 in multipli-
cation. 1.1.13 Singular Matrices. A matrix is called singular if
its determinant is zero. Singular matrices do not have an
Example: " # inverse.
1 0
I=
0 1 Example: Consider the matrix
" #
Not all matrices have inverses. A matrix with an inverse 2 2
B=
is called invertible or non-singular. 1 1

The determinant of B is 2 · 1 − 2 · 1 = 0, making B

Basics of Subsidiary Mathematics By Khalid M +256 754857296 khalidhkmsh@gmail.com 2


1.2. INDICES, SURDS AND LOGARITHMS 1. ALGEBRA

singular. Since its determinant is zero, B does not have Example: For the given system of equations, the deter-
an inverse. minant of matrix A is 2 · 3 − (−1) · 1 = 7.

Calculating determinants, finding inverses, and under- det(Ax ) det(Ay )


x= , y=
standing singular matrices are crucial concepts in matrix det(A) det(A)
algebra. They play a significant role in solving systems
of equations and various applications. Where Ax is obtained by replacing the first column of A
with matrix B and Ay is obtained by replacing the second
1.1.14 Real-world Applications. Matrices are used
column of A with matrix B.
in various real-world applications, including computer
graphics, transformations, population modeling, and solv- " # " #
ing systems of linear equations. 5 −1 2 5
Ax = , Ay =
7 3 1 7
1.1.15 Solving Linear Systems in x and y. There are
various methods to solve systems of linear equations in- Calculate the determinants and solve for x and y.
volving two variables x and y. Two common methods
are the adjoint method and Cramer’s rule.
1.2 INDICES, SURDS AND
1.1.16 Adjoint Method. Consider the system of equa-
tions: LOGARITHMS
2x − y = 5
x + 3y = 7 1.2.1 Indices. Indices, also known as exponents or pow-
ers, represent the number of times a base is multiplied by
We can write this system in matrix form as AX = B,
itself.
where:
" # " # " # Example: 23 means 2 raised to the power of 3, which is
2 −1 x 5 2 · 2 · 2 = 8.
A= , X= , B=
1 3 y 7
Some important rules of indices are:
To solve for X, we can use the adjoint method:
ˆ am · an = am+n
am
X = A−1 B ˆ an
= am−n

ˆ (am )n = am·n
Example:
" # " # Indices, or exponents, come in various types: positive,
1 3 1 5 negative, and fractional.
A−1 = , B=
2 · 3 − (−1) · 1 −1 2 7
1.2.2 Positive Indices. Positive indices represent the
" #
3 number of times a base is multiplied by itself.
X = A−1 B =
1 Example: 23 means 2 raised to the power of 3, which is
2 · 2 · 2 = 8.
Thus, the solution is x = 3 and y = 1 using the adjoint
method. 1.2.3 Negative Indices. Negative indices indicate tak-
ing the reciprocal (inverse) of the base raised to the pos-
1.1.17 Cramer’s Rule. Cramer’s rule provides a solution
itive index.
for each variable by calculating determinants.
Example: 5−2 means 1
52
, which is 1
25
.

Basics of Subsidiary Mathematics By Khalid M +256 754857296 khalidhkmsh@gmail.com 3


1.2. INDICES, SURDS AND LOGARITHMS 1. ALGEBRA

1.2.4 Fractional Indices. Fractional indices, or roots, So, the solutions to the equation 22x − 5 · 2x + 4 = 0 are
represent the inverse operation of raising a number to an x = 2 and x = 0.
exponent.
1.2.5 Logarithms. Logarithms are the inverse opera-
1/2
Example: 16 means the square root of 16, which is tions of exponentiation. The logarithm of a number to a
4. specific base gives the power to which the base must be
raised to obtain that number.
Some important rules for fractional indices include:
√ Example: log10 100 = 2 because 102 = 100.
ˆ n am = am/n
Some logarithmic rules include:
√ √ √
ˆ na· b= a·b
n n

ˆ loga (xy) = loga x + loga y


If we have the equality (2/3)−1/2 = (3/2)1/2 involving
reciprocal and fractional indices.  
ˆ loga x
y
= loga x − loga y
Left-hand side:
 −1/2 ˆ loga xn = n · loga x
2 1
=
3 2 1/2

3 1.2.6 Change of Base in Logarithm. The change of
base formula in logarithms is a useful technique when you
Right-hand side:
need to evaluate logarithms in bases other than those
available on your calculator.
r
1/2 3
(3/2) =
2 Change of Base Formula: For any positive numbers
a, b, and c where a 6= 1 and b 6= 1, the change of base
Example: Given the equation 22x − 5 · 2x + 4 = 0, let’s
formula states:
use substitution to make it a quadratic equation. logc b
loga b =
logc a
Substitution: Let y = 2x , then the equation becomes:
This formula allows you to evaluate logarithms using a
2
y − 5y + 4 = 0 base (c) that your calculator supports.

Example: Evaluate log3 10 using the change of base for-


This is a quadratic equation that can be factored: mula with base c = 10:

(y − 4)(y − 1) = 0 log10 10 1
log3 10 = =
log10 3 log10 3

So, y = 4 or y = 1.
Example 1: Solve for x in the equation 3x = 27.
Since y = 2x , we have two cases to consider:
3x = 27
x
Case 1: y = 4 (which corresponds to 2 = 4)
log3 (3x ) = log3 27
2x = 4 ⇒ x = log2 4 ⇒ x = 2 x = log3 27
x=3
Case 2: y = 1 (which corresponds to 2x = 1)
The logarithmic transformation converts the exponential
x
2 = 1 ⇒ x = log2 1 ⇒ x = 0 equation into a linear one.

Basics of Subsidiary Mathematics By Khalid M +256 754857296 khalidhkmsh@gmail.com 4


1.2. INDICES, SURDS AND LOGARITHMS 1. ALGEBRA

√ √
Example 2: Solve for y in the equation 2 · 5y = 50. Example: Simplify 2· 3.
√ √ √
2 · 5y = 50 2· 3= 6
log5 (2 · 5y ) = log5 50
log5 2 + log5 (5y ) = log5 50 1.2.10 Rationalization of Surds. Rationalizing surds
log5 2 + y = log5 50 involves eliminating radicals from the denominator of a
y = log5 50 − log5 2 fraction. This is done to simplify expressions and make
50 them more manageable.
y = log5
2 1.2.11 Rationalizing with Conjugates. One common
y = log5 25 method is to multiply both the numerator and denomi-
y=2 nator of a fraction by the conjugate of the denominator.

Example 1: Rationalize the denominator of √1 .


2
1.2.7 Surds. A surd is an expression that includes square
roots or other roots that cannot be simplified to rational √ √
1 2 2
numbers. √ ·√ =
2 2 2

Example: 2 is a surd because it cannot be simplified
Example 2: Rationalize the denominator of √3 .
further. 5+2

Operations with surds follow rules like: √ √


3 5−2 3( 5 − 2)
√ ·√ =
5+2 5−2 5−4

√ √ √ = 3( 5 − 2)
ˆ a· b= a·b

√ 1.2.12 Rationalizing with Higher Roots. For higher


ˆ a pa
√ =
b b roots, like cube roots or higher, the same method applies.
Multiply by the conjugate to eliminate the radical in the
1.2.8 Addition and Subtraction. When adding or sub- denominator.
tracting surds, the terms inside the radical must be the Example 3: Rationalize the denominator of 1
3 .

3
same.
√ √
√ √ 1 3
9 3
9
Example: Simplify 2 3 + 3 3. √
3
· √
3
=
3 9 3
√ √ √ √

2 3 + 3 3 = (2 + 3) 3 = 5 3 √2+ √2
Let’s rationalize the expression 5− 3
using the conju-
√ √ gate method.
Example: Simplify 5 2 − 2 2.
√ √ √ √ √ √ √ √ √ √
5 2 − 2 2 = (5 − 2) 2 = 3 2 2+ 2 5+ 3 (2 + 2)( 5 + 3)
√ √ ·√ √ = √ √ √ √
5− 3 5+ 3 ( 5 − 3)( 5 + 3)
√ √ √ √
1.2.9 Multiplication. When multiplying surds, you can 2 5 + 2 3 + 10 + 6
=
use the distributive property. √ √5 − 3√ √
√ √ 2 5 + 2 3 + 10 + 6
Example: Simplify 2 5 · 3 5. =
2√ √
√ √ √ √ 10 + 6
2 5 · 3 5 = 6 · 5 = 30 = 5+ 3+
2

Basics of Subsidiary Mathematics By Khalid M +256 754857296 khalidhkmsh@gmail.com 5


1.3. SEQUENCE, SERIES, SIGMA NOTATION 1. ALGEBRA

Un
1.2.13 Practice and Exercises. Work on problems in- d) Un+1 = Un+2
where U1 = 2
volving indices, surds, and logarithms to reinforce con-
cepts. Solution (a) We substitute for to find the first three terms

1.2.14 Real-world Applications. Indices, surds, and ˆ For n = 1, U1 = 1(1 + 1) = 2


logarithms are used in various fields, including science,
ˆ For n = 2, U1 = 1(1 + 1) = 6
engineering, finance, and computer science.
ˆ For n = 3, U1 = 1(1 + 1) = 12

1.3 SEQUENCE, SERIES, SIGMA (b) Since the first term is given Then when

NOTATION ˆ For n = 1, U2 = 2 − 5
U1
= −3

ˆ For n = 2, U3 = 2 − 5
U2
= 3 23
There are many sequences found in nature. A mathemati-
cian known as Leonardo of Pisa (Fibonacci) [1170-1250] (c) and (d) left as an exercise
first discovered the sequence of numbers which would be-
Example 2: Write down the nth term of each sequence
come known as the Fibonacci Sequence. This sequence
can be used to describe can be used to describe the spiral a) 1, 4, 9, 16, · · ·
which appears in sea shells, the arrangement of seeds in a
b) 3, 7, 11, 15, 16, · · ·
flowering plant and many other sequences found through-
out nature. c) 2, 5, 10, 17, 26, · · ·
1.3.1 Sequences. When a set of numbers follow a pat- d) 1 1 1 1 1
, , , , ,···
2 3 4 5 6
tern and there is a clear rule for finding the next number
in the pattern then we have a sequence. The following Solution (a) nth term is n2 because squaring counting
are examples of sequences; number results into square numbers

ˆ 2, 4, 6, 8, · · · (1)2 , (2)2 , (3)2 , (4)2 , · · · (n)2

ˆ 1, 4, 9, 15, 25,· · ·
(b)
ˆ 2, -4, 8, -16, 32,· · · 3, 7, 11, 15, 17, · · · 4n − 1

4n becomes the difference between each term −1 be-


Each number in sequence is called is called term of the
comes the 1st term 4(1) − 1 = 3
equation U1 is the first term of sequence U2 is the second
term of the sequence Un is the nth term of the sequence (c), (d) and (e) left as an exercise
Un can be used to describe any term in the sequence
1.3.2 Series. A series is formed when the terms of a
Un+1 is the term after Un
sequence are added e.g.,
Example 1: Write down the three terms of each se-
quence 12 + 9 + 6 + 3 + · · ·

a) Un = n(n + 1) A series is finite if it stops after a countable number of


b) Un+1 = 2 − 5
where U1 = 1 terms e.g.,
Un

c) Un = (−1)n 7n2 5 + 9 + 13 + 17 + 21 + 25

Basics of Subsidiary Mathematics By Khalid M +256 754857296 khalidhkmsh@gmail.com 6


1.4. ARITHMETIC PROGRESSIONS APS AND GEOMETRIC PROGRESSIONS GPS 1. ALGEBRA

is a finite series, it has 6 terms. A series is infinite if it 1.4.3 Generating an A.P.. To generate an A.P., start
continues indefinitely e.g., with an initial term (a1 ) and add the common difference
(d) successively to each term.
5 + 9 + 13 + 17 + 21 + · · ·
Example

1.3.3 Sigma Notation Σ. The symbol Σ was intro- Generate an A.P. with a1 = 3 and d = 2.
duced by Leonard Euler to denote the sum of series. It The A.P. will be: 3, 5, 7, 9, 11, ...
is useful to have a short way of writing expressions like
1.4.4 Finding the nth Term of an A.P.. The nth term
12 +22 +· · ·+n2 This can be done by writing Σm2 which
(an ) of an A.P. can be determined using the formula:
means the sum of all terms like m2 Thus

an = a1 + (n − 1)d
Σn1 m2 = 12 + 22 + 32 + · · · + n2

and Example

Find the 10th term of the A.P. with a1 = 3 and d = 2.


Σ5r=2 r(r + 2) = 2(4) + 3(5) + 4(6) + 5(7)
Using the formula:
Example 2: Write out the series defined by the following
a10 = 3 + (10 − 1) · 2 = 3 + 18 = 21

a) Σ4m=1 m3
So, the 10th term is 21.
b) Σnk=2 (k 2 + k)
1.4.5 Determining the Sum of the First n Terms.
1 To find the sum of the first n terms (Sn ) of an A.P., you
c) Σ5r=1 r(r+1)
can use the formula:
d) Σnq=1 (−1)q q 2 n
Sn = [2a1 + (n − 1)d]
2
e) Σnj=1 (j − 4)(j + 4)
Example

Find the sum of the first 5 terms of the A.P. with a1 = 3


1.4 Arithmetic Progressions APs
and d = 2.
and Geometric Progressions Using the formula:
GPs 5 5 5
S5 = [2 · 3 + (5 − 1) · 2] = [6 + 8] = · 14 = 35
2 2 2
1.4.1 Arithmetic Progressions (A.P.). An Arithmetic
Progression (A.P.) is a sequence of numbers in which the So, the sum of the first 5 terms is 35.
difference between any two consecutive terms is constant. Example 3.2.2 [Guided Question]
This constant difference is called the common difference,
1.4.6 (a). Find the sum of 8 terms of an AP whose first
denoted as d.
term is 37 and common difference is −4.
1.4.2 Identifying A.P.s. To identify an A.P., check if
the difference between consecutive terms is constant. If
it’s constant, then the sequence is an A.P.

Basics of Subsidiary Mathematics By Khalid M +256 754857296 khalidhkmsh@gmail.com 7


1.4. ARITHMETIC PROGRESSIONS APS AND GEOMETRIC PROGRESSIONS GPS 1. ALGEBRA

Using Theorem 3.2.1: 1.4.10 (b). Jamila starts a part-time job on a salary of
shs. 9000 per year, and this increases by an annual in-
n
Sn = [2a + (n − 1)d] crement of shs. 1000. Assuming that, apart from the
2
increment, Jamila’s salary does not increase, find:
Given: n = 8, a = 37, d = −4
(i) Her salary in the 12th year.
8 (ii) The length of time she has been working when her
S8 = [2·37+(8−1)·(−4)] = 4[74−28] = 4[46] = 184
2
total earnings are shs. 100,000.

Answer: S8 = 184 1.4.11 Geometric Progressions. A Geometric Progres-


sion (G.P.) is a sequence of numbers in which each term is
1.4.7 (b). In an AP, U10 = 3 and S6 = 76.5. Find a, d,
found by multiplying the previous term by a fixed, nonzero
and the value of n for which Sn < 0.
number called the common ratio, denoted as r.
Using Theorem 3.1.1:
1.4.12 Identifying G.P.s. To identify a G.P., check if
Uk = a + (k − 1)d for n = 10 the ratio of any two consecutive terms is constant. If it’s
constant, then the sequence is a G.P.

Obtain the first algebraic equation. 1.4.13 Characteristics of a G.P.. The characteristics
of a G.P. include: - A fixed common ratio (r). - The first
Also, for m, another equation using Theorem 3.2.1:
term (a1 ). - The general term an given by an = a1 ·r(n−1) .
n
Sn = [2a + (n − 1)d] for n = 6 1.4.14 Generating a G.P.. To generate a G.P., start
2
with an initial term (a1 ) and multiply it by the common
Solve the two equations simultaneously to obtain a = ratio (r) successively to each term.
16.5 and d = −1.5. Example
n
Suppose Sn = 0, then Sn = 2
[2a + (n − 1)d] = 0. Generate a G.P. with a1 = 2 and r = 12 .
Substitute in the values a and d to obtain n. [Answer: The G.P. will be: 2, 1, 0.5, 0.25, ...
n = 23]
1.4.15 Sum of n Terms of a G.P.. To find the sum of
1.4.8 (c). The sum of the first ten terms of an AP is the first n terms (Sn ) of a G.P., you can use the formula:
120, and the sum of the first twenty terms is 840. Find
the sum of the first thirty terms. a1 (1 − rn )
Sn =
1−r
Similar to (a) and (b), use Theorem 3.2.1 to generate
two equations and solve. [Answer: 1575d] Example:
Example 3.2.3 [Real Life Experiences] Consider a G.P. with a1 = 3 and r = 12 . Find the sum of
1.4.9 (a). A student reading a book finds that he reads the first 4 terms.
faster as he gets into the subject. He reads 19 pages
on the first day, and his rate of reading then goes up by
3 pages each day. How long does he take to finish the
book?

Using Theorem 3.2.1, given that Sn = 426, d = 3, and


a = 19. [Answer: n = 12 days]

Basics of Subsidiary Mathematics By Khalid M +256 754857296 khalidhkmsh@gmail.com 8


1.4. ARITHMETIC PROGRESSIONS APS AND GEOMETRIC PROGRESSIONS GPS 1. ALGEBRA

Using the formula:


25 6 25 6 25 5
a = 12 6 , r= ÷ 12 6 = × =
5 5 12 2
 
1 4
3 1− 2
5 5
S4 =
1 − 12  5 ! 
1
 25 5 5
3 1 − 16 S5 = 12 6 −1 − 1 = 30.93
= 1 5
2 2
2
15

3
= 16 1.4.18 (b). A geometric progression in which a = 27
1
2 and r = 23 .
45
=
8 Solution:
= 5.625 703
Use Theorem 3.5.1 (i). [Answer: 1
]

So, the sum of the first 4 terms of this G.P. is 5.625. Example 3.5.2

1.4.16 Sum to Infinity of a G.P.. The sum to infinity A geometric progression has first terms 27 and a common
(S∞ ) of a G.P. exists if −1 < r < 1. It can be calculated ratio 43 . Find the least number n of terms the series can
using the formula: have if its sum exceeds 550.

a1 Solution:
S∞ =
1−r
Given: a = 27, r = 34 .

Where: - S∞ is the sum to infinity. - a1 is the first term Apply Theorem 3.5.1 appropriately.
of the G.P. - r is the common ratio.
Sn > 550
Example:
Remember to apply logarithms.
1
Consider a G.P. with a1 = 2 and r = 3
. Find the sum to
Answer: n = 8
infinity.
Example 3.5.3
Using the formula:
1.4.19 (a). In a GP, U3 = 32 and U6 = 4. Find a and
2
S∞ = 1
r, and the sum of the first eight terms.
1− 3
2 Solution:
= 2
3 Using Theorem 3.5.1, Un = arn−1 .
=3
Formulate two equations in terms of a and r.

So, the sum to infinity of this G.P. is 3. Example 3.5.1 Divide the two equations to get r3 = 18 , which implies
r = 12 .
1.4.17 (a). Find the sum of the first five (S5 ) terms of:
Use one of the equations to get a = 128.
25
12 6 + 3 + ... Apply Theorem 3.5.1: S8 = 255.
5
1.4.20 (b). In a GP, the sum of the second and third
Solution:
term is 6, and the sum of the third and fourth terms is
Using Theorem 3.6.1 (ii): −12. Find the first term and common ratio.

Solution:

Basics of Subsidiary Mathematics By Khalid M +256 754857296 khalidhkmsh@gmail.com 9


1.4. ARITHMETIC PROGRESSIONS APS AND GEOMETRIC PROGRESSIONS GPS 1. ALGEBRA

Using Theorem 3.5.1, formulate two equations and factor ∞


out the common term in each.
X 1 1 1 1
n
= 1 + + + + ...
n=1
2 2 4 8
Divide the two equations to find r = −2.
Solution:
Obtain a using any of the equations: a = 3.
The partial sums:
3.6 Geometric Mean
First partial sum S1 = 1.
If some number y is inserted between a pair of numbers p
1
and q such that p, y, and q are in geometric progression, Second partial sum S2 = 1 + 2
= 23 .
then y is the geometric mean of p and q. It follows that
√ Third partial sum S3 = 1 + 12 + 1
4
= 74 .
r = yp = yq , cross-multiplying, y 2 = pq, and y = pq.
Since the partial sums are getting closer to a specific
3.7 Sum to Infinity of a GP
value (in this case, 1), the series is convergent.
Consider the infinite geometric progression:
3.9.2 Example

Show that the following series will diverge:

a) ∞ n
P
18 + 1.8 + 0.18 + 0.018 + 0.0018 + 0.00018 + . . . n=0 (−2)

b) ∞ n
P
n=0 2
1
The first term is 18, and the common ratio is 10
.
Solution:
For this GP:
a) The series ∞ n
P
n=0 (−2) is a geometric series with a
S2 = 18 + 18 = 19.8 common ratio of −2. The partial sums are as follows:
S3 = 18 + 1.8 + 0.018 = 19.98 First partial sum S1 = 1.
S4 = 19.998 Second partial sum S2 = 1 − 2 = −1.
S5 = 19.9998 Third partial sum S3 = 1 − 2 + 4 = 3.
Clearly, Sn approaches the value 20 as n approaches Since the partial sums are not getting close to a specific
infinity. This is written as Sn → 20 as n → ∞ or value, the series is divergent.
limn→∞ Sn = 20.
b) The series ∞ n
P
n=0 2 is also a geometric series with a
For a general GP, a + ar + ar2 + ar3 + . . . and if common ratio of 2. The partial sums are as follows:
−1 < r < 1 (i.e., |r| < 1), then:
First partial sum S1 = 1.

a Second partial sum S2 = 1 + 2 = 3.


S∞ =
1−r
Third partial sum S3 = 1 + 2 + 4 = 7.
3.8 Convergence and Divergence of Series
Since the partial sums ar
A convergent series is a series whose partial sums tend to
a specific number, also called a limit. A divergent series
is a series whose partial sums do not approach a limit.

3.8.1 Example

Show that the given series is convergent:

Basics of Subsidiary Mathematics By Khalid M +256 754857296 khalidhkmsh@gmail.com 10


1.5. FACTORIAL NOTATION, PERMUTATION AND COMBINATIONS 1. ALGEBRA

1.5 FACTORIAL NOTATION, 1.6 Examples


PERMUTATION AND COM-
Example 1.3 Write 9! in factorial notation.
BINATIONS
9! = 9 · 8 · 7 · 6 · 5 · 4 · 3 · 2 · 1

Example 1.4 Express 10 · 9 · 8 in factorial notation.

10 · 9 · 8 = 10!

In many calculations, you will encounter the symbol ”!,”


1.7 Exercises
which you may not have encountered before in your math-
ematics classes. This symbol represents the concept of a 1.7.1 Exercise 1.1. (1) Evaluate:
factorial.
(a) 3!
1.5.1 Factorial. Factorial means that we multiply all
integers less than or equal to the chosen number. For (b) 4!
example, 5! means we multiply all numbers less than or
equal to 5. We start with 5 and keep multiplying it with (c) 5!
the numbers as we count down all the way to 1.
(d) 6!
Definition The factorial n! is defined by:
(e) 7!
n! = n · (n − 1) · (n − 2) · . . . · 2 · 1
(2) Express in factorial notation:
Where n = 0, 1, 2, 3, . . .. For n = 0, 0! = 1.
(a) 6 · 5
Note: Many calculators have a special key labeled [x!].
For small values of x, the calculator provides the exact (b) 9 · 8 · 7
value. However, as the values of x increase, the numbers
grow rapidly. For values around 14 and on wards, only (c) 12 · 11 · 10
approximate values in standard form can be displayed.
(d) 15 · 14 · 13 · 12
1.5.2 Dividing Factorials. When dividing factorials,
similar terms in the denominator and the numerator can-
cel out, and we simplify the remaining answer. 1.8 Permutations
4!
Example 1.1 Evaluate using
2!
1.8.1 Introduction. Consider three letters X, Y , and Z.
7!
Example 1.2 Evaluate using If these letters are written in a row one after the other,
5!
1.5.3 Expressing in Factorial Notation. In this sec- there are six different ways of arranging them:
tion, we can either multiply a number by a factorial in
XY Z, XZY, Y XZ, Y ZX, ZXY, ZY X
both the numerator and the denominator or expand the
numerator.

Basics of Subsidiary Mathematics By Khalid M +256 754857296 khalidhkmsh@gmail.com 11


1.8. PERMUTATIONS 1. ALGEBRA

Each of these arrangements is a possible permutation of 1. (a) The number of possible arrangements of the
the letters X, Y , Z, so there are six permutations alto- books.
gether.
2. (b) If these books, 3 are red-covered books, in how
Definition 1.2.1 A permutation is an arrangement of many arrangements can these be kept together?
objects without repetition, and order is important.
Solution:
ˆ Permutations that arrange objects in a line are
called linear permutations. 1. (a) Number of books n = 12 (with no restriction).
Possible arrangements: n! = 12 · 11 · 10 · . . . · 1 =
ˆ Permutations that arrange objects in a circle are 479001600 ways.
called circular permutations.
2. (b) Three red-covered books kept together. These
three can be arranged by reversing this order in 3!
In practice, the order may be in space, such as from left
ways among themselves. If these 3 are together,
to right in a row, or it may be in time, such as reaching
then there are 10 to be arranged. Therefore, there
the winning spot in a race.
are 10! · 3! ways.
In the above scenario, there are 6 ways of arranging the
3 letters: 3! = 3 · 2 · 1 = 6. 10! · 3! = 21772800 ways

Example 1.2.1 Five children are to be seated on a bench.


Example 1.2.3 Seven girls and two boys are to sit on a
bench. In how many ways can they arrange themselves
1. (a) How many ways can the children be seated?
so that the boys do not sit next to each other? [Ans
2. (b) How many arrangements are possible if the 282240] Hint: No Restriction - With Restriction
youngest sits at the left end of the bench? and Subtract

1.8.2 Circular Arrangements. This is the total num-


Solution:
ber of ways in which objects can be arranged ”in a circle”
order. For example, the guests around a table at a din-
1. (a) Since there are five (5) children, the first place
ner party with circular arrangements, the relative position
can be filled in 5 possible ways, and 4 left. The
of items being arranged is important. Therefore, we may
second place in 4 possible ways and 3 left, and so
consider one object to be fixed and others can be arranged
on. The fifth place is filled in 1 way.
about it in (n − 1) ways.

5 · 4 · 3 · 2 · 1 = 5! = 120 ways Two cases arise:

1. Clockwise and anticlockwise orders are different


2. (b) If the youngest sits on the left, then there are
(e.g., when dealing with people). Then Pn =
4 places left to be filled by 4 people. The second
(n − 1)!, where Pn represents circular permutation
place is filled in 4 possible ways, and 3 left, and so
and n is the number of objects.
on. The last one fills the last place.
2. Clockwise and anticlockwise orders are the same
1 · 4 · 3 · 2 · 1 = 1 · 4! = 24 ways (e.g., beads). For example, Pn = (n−1)! = 12 (n−1)!
2!

Example 1.2.2 There are 12 textbooks on the shelf to Example 1.2.4 In how many ways can 8 people sit at a
be arranged. Find: round table?

Basics of Subsidiary Mathematics By Khalid M +256 754857296 khalidhkmsh@gmail.com 12


1.8. PERMUTATIONS 1. ALGEBRA

1.8.3 Solution. Since 8 people are identical, then Pn = 1.8.6 Arrangements of Objects Selected from a
(n − 1)! Group. Suppose we wish to arrange r objects chosen
from n unlike objects. The permutation of r objects from
P8 = (8 − 1)! = 7! = 7 · 6 · 5 · 4 · 3 · 2 · 1 = 5040 ways n unlike objects is given by:

Example 1.2.5 Nine beads, all of different colors, are Pnr = n(n − 1)(n − 2)(n − 3) . . . (n − r + 1)
to be arranged on a circular wire. How many different
Expanding in factorial notation:
arrangements are possible if two arrangements are not
considered different? n!
Pnr =
(n − r)!
1.8.4 Solution.

1 For Example: In how many ways can two different let-


Pn = (n − 1)!
2 ters be chosen from A, B, C, D, and E? Solution The
1 1 1 first letter can be chosen in 5 ways, and the second letter
P9 = (9−1)! = (8!) = ·8·7·6·5·4·3·2·1 = 20160 ways
2 2 2 in 4 ways. Hence, there are 5 · 4 = 20 possible ways as
seen: AB, AC, AD, AE, BC, BD, BE, CD, CE, DE.
1.8.5 Mutually Exclusive Situations. When two sit-
uations X and Y are mutually exclusive, if situation X Or:
5! 5 · 4 · 3!
occurs, situation Y cannot occur, and vice versa. In this P52 = = = 5 · 4 = 20
(5 − 2)! 3!
case, the number of permutations of either situation X
or situation Y occurring can be obtained by adding the 1.8.7 Arrangements of Like and Unlike Things.
number of permutations of X to that of Y .

Example 1.2.6 How many different four-digit numbers ˆ The number of permutations of n things taken all
can be formed from the figures 1, 2, 5, 6 if each figure is at a time, when repetition of objects is allowed, is
used only once? How many of these numbers: nn .

1. (a) end with 6? ˆ The number of permutations of n objects taking r


at a time, when repetition of objects is allowed, is
2. (b) end with 1?
nr .
3. (c) end with 6 or 1?
1.8.8 Example 2. How many different three-digit num-
Solution The first digit can be chosen in 4 ways, the
bers can be formed from the figures 3, 5, 6, and 7 if
second in 3 ways, and so on. Thus, there are 4! = 24
repetitions:
different ways.

1. (a) The last digit is 6, and it must be chosen in one 1. (a) are allowed?
way. The first digit can be chosen in 3 ways, the
second digit in 2 ways, the third digit in one way. 2. (b) are not allowed?
Total: 3 · 2 · 1 · 1 = 6 ways.
Solution
2. (b) Using similar reasoning, there are 6 ways.

3. (c) Since the numbers cannot end with both 6 and 1. (a) Using n = 4 and r = 3 with repetition allowed:
1, they are mutually exclusive situations. Thus,
6 + 6 = 12 of the numbers end with 6 or 1. 43 = 64 different numbers

Basics of Subsidiary Mathematics By Khalid M +256 754857296 khalidhkmsh@gmail.com 13


1.8. PERMUTATIONS 1. ALGEBRA

2. (b) Using n = 4 and r = 3 with no repetition 1.8.11 Example 5. Find the number of ways in which
allowed: the letters of ISOSCELES can be arranged if the two Es
are separated.
4!
4P 3 = = 24 different numbers
(4 − 3)! Solution [left as a trial question]

ˆ The number of permutations of n objects, of which


p are of one kind, q are of the second kind, and so ˆ Identify the similar letters.
n!
on, is p!q!r!... .

1.8.9 Example 3. How many ways can 4 red, 3 yellow, ˆ Use n!


p!q!r!...
.
and 2 green dresses be arranged in a row if dresses of the
same color are indistinguishable? ˆ Repeat the same when Es are together.
Solution There are 4 + 3 + 2 = 9 dresses. The number
of arrangements is: ˆ Subtract the results. Answer: [23,520]
9!
= 1260
4!3!2!
1.8.12 Combinations.
1.8.10 Example 4. 1.8.13 Introduction. In many occasions, we are not
interested in arranging but only in selecting ”r” objects
1. (a) In how many different ways can the letters of
from ”n” given objects. A combination is a selection of
the word ALGEBRA be arranged in a row?
some or all of a number of different objects where the
2. (b) In how many of these arrangements are the two order of selection is not to be considered.
As together?
We earlier saw that 2 letters can be arranged in a row
3. (c) In how many of these arrangements are the two from ABC in 6 ways as shown:
As not together?

Solution
AB ACBC

1. (a) There are 7 letters, including two As. There- BA CACB


fore, the total number of arrangements is:
But the selection A and B is the same as B and A. The
7!
= 2520 number of selections becomes 3. In general, we know
2!
that arrangements of n things taken ”r” at a time is:
2. (b) If the As are kept together, there are effectively
6 letters to arrange: n!
nP r =
(n − r)!
LGEBRA
But each selection of ”r” things can be arranged in r!
Since the arrangement of AA is the same, AA is ways, so each solution is repeated r! times in the nPr
taken as 1! Total arrangements: 6! × 1! = 720 arrangements. Hence, the number of selections is:

3. (c) Number of arrangements when As are not to- n!


gether: 2520 − 720 = 1800 nCr =
(n − r)!r!

Basics of Subsidiary Mathematics By Khalid M +256 754857296 khalidhkmsh@gmail.com 14


1.8. PERMUTATIONS 1. ALGEBRA

1.8.14 Theorem 1.3.1. The number of possible com- 1.8.19 Solution. (i) Find using Theorem 1.3.1.
binations of ”n” different objects taken ”r” at a time is
(ii) Number of choosing 3 boys from 7 boys.
given by:
(iii) Number of choosing 2 girls from 5 girls.

nCr (iv) Multiply the two results since the choice of 3 boys
and the choice of 2 girls are independent.
Examples 1.3.1: Using Theorem 1.3.1, find the values:
1.8.20 Answer. 350

1. (a) 8C3 1.8.21 Mutually Exclusive Combinations. Example


1.3.5 A group consists of 4 boys and 7 girls. In how
2. (b) 8C5
many ways can a team be selected if it is to contain:
3. (c) 10C6
1. (a) no boys
4. (d) 10C4

2. (b) at least one member of each sex


Example 1.3.2 How many selections of 4 letters can be
made from 6 letters: a, b, c, d, e, and f?
3. (c) at least 3 boys
1.8.15 Solution. Number of selections is nCr:
1.8.22 Solution. (a) No boys selected, so the team is
n = 6, r=4 chosen from 7 girls, choosing 5 girls from 7:

6! 6! 7C5 = 21
6C4 = = = 15
(6 − 4)!4! 2!4!
(b) At least one member of each (fill in the missing
1.8.16 Example 1.3.3. (a) Find the number of different spaces):
selections of 4 letters that can be made from the letters
of the word SPHERICAL.

(b) How many of these selections do not contain a vowel? 4 Boys 7 Girls Possible Selections
1.8.17 Solution. (a) n = 9 (All different letters). Num- 14 4C1 × 7C4
9!
ber of selections of 4 letters: 9C4 = 5!4! = 126. 23 4C2 × 7C3
(b) There are 3 vowels: E, I, A. Each word either does 32 4C3 × 7C2
or doesn’t contain a vowel. If these selections are not to 41 4C4 × 7C1
contain vowels, then 4 letters are selected from 6: Total 441
6!
6C4 = 2!4!
= 15
Since these possible selections are mutually exclusive, we
1.8.18 Example 1.3.4 [Independent Operations].
add all of them and get 441.
How many different committees, each consisting of 3 boys
and 2 girls, can be chosen from 7 boys and 5 girls? (c) Left as an exercise.

CONTANT FOR COMPLETE BOOK

Basics of Subsidiary Mathematics By Khalid M +256 754857296 khalidhkmsh@gmail.com 15


2. LINEAR PROGRAMMING
2.1 Introduction model, we start by defining the decision variables
that represent the choices we want to make. These
Linear Programming is like a problem-solving tool for variables should fully describe the decisions we need
managers. It’s a math method used to figure out the best to make. For example consider the bakery scenario,
way to use resources like money, people, materials, ma- we want to determine how many units of two dif-
chines, and more, to achieve a specific goal when there ferent products to bake each day. So we let x1
are different ways to use these resources. represents the number of cakes to bake daily.

Linear programming problems come in different types, but x2 represents the number of loaves of bread to bake
two common ones are: daily.

The Product-Mix Problem: This problem occurs when 2. Objective Function: In every linear programming
you have multiple products, and they all need resources problem, the decision maker aims to either maxi-
that are limited. The goal is to figure out which products mize (usually revenue or profit) or minimize (usually
to make and how much of each to produce or sell so that costs) a specific function of the decision variables.
you can make the most profit, capture the most market This function is called the objective function.
share, or generate the highest sales revenue. The bakery’s objective is to maximize profit, which
The Blending Problem: This problem is about finding depends on the sales of cakes and bread. Let’s as-
the best combination of available ingredients to create a sume the bakery earns Shs. 10 in profit for each
specific quantity of a product while meeting strict quality cake sold (x1 ) and Shs. 5 in profit for each loaf of
requirements. ”Best blend” here means the combination bread sold (x2 ). The objective function, represent-
that costs the least among the necessary inputs. ing total daily profit, is:

A linear programming problem (LP) is a math problem Maximize Z = 10x1 + 5x2


where we try to do these things:
The coefficients 10 and 5 are the objective function
ˆ We want to make a linear equation with the things coefficients, representing the contribution to profit
we’re deciding. We either want it to be as big as from each product.
possible or as small as possible. We call this equa-
3. Constraints: As we increase the values of x1
tion the ”main goal.”
and x2 , The baker’s objective function (like profit)
ˆ The numbers we choose for our decisions have to grows larger. This implies that if He could choose
follow some rules. These rules are like math sen- any values for x1 and x2 , the bakery could make a
tences that are straight lines or use less than or very large profit by selecting extremely high values
equal to signs. for these variables. However, there are limits to
the values of x1 and x2 imposed by three specific
ˆ Each decision we make must be either a number
constraints:
that’s not negative (like 0 or more) or it can be
any number, positive or negative. Oven Capacity Constraint: The bakery can only
use the oven for a limited number of hours each
2.1.1 Formulation of Linear Programming Model. day. Let’s say the cakes take 1 hour to bake, and
the bread takes 0.5 hours. The total oven hours
1. Decision Variables: In a linear programming are limited to, for example, 5 hours.

Basics of Subsidiary Mathematics By Khalid M +256 754857296 khalidhkmsh@gmail.com 16


2.2. INEQUALITIES AND GRAPHS 2. LINEAR PROGRAMMING

a. y ≥ 4x−7 The line y = 4x−7 will form the boundary


x1 + 0.5x2 ≤ 5 (Oven constraint) of the region. A solid line is drawn as the inequality con-
tains a ’≥’ sign which means that points on the boundary
Ingredient Constraint: There’s a limited supply of are included
ingredients available daily. For example, the cakes
Next, select a point such as (3, 2). (It does not matter
require 4 cups of flour and the bread 2 cups. Let’s
on which side of the line the point lies.)
say there are 20 cups of flour available daily.
If the values, x = 3 and y = 2, are substituted into the
4x1 + 2x2 ≤ 20 (Flour constraint) inequality, we obtain 2 ≥ 5 This statement is clearly false
and will also be false for any point on that side of the line.
Time Constraint: The bakery has a limited number Therefore the other side of the line should be shaded, as
of work hours available each day. Suppose there are shown.
8 work hours available daily.

x1 + x2 ≤ 8 (Time constraint)

These constraints ensure that the bakery can’t pro-


duce an unlimited number of cakes and bread,
and they help keep the production within realistic
bounds.

2.2 Inequalities and Graphs

When an inequality involves two variables, the inequality


b. x + 2y < 10 The line x + 2y = 10 will form the
can be represented by a region on a graph.
boundary of the region, but will not itself be included in
For example Represent the following inequality on the the region. To show this, the line should be drawn as a
graph and shade the wanted region dotted line.

Now 3 points on the line can be calculated, for example


a) y ≥ 4x − 7
x 0 2 4
b) x + 2y < 10
y 5 4 3

c) x + y ≥ 4
(0, 5), (2, 4) and (4, 3). are the points where the line
d) y > x − 2 passes.

Next, a point on one side of the line is selected, for


e) 2x + y ≥ 5 example (2,3), where x = 2 and y = 3. Substitut-
ing these values for x and y into the inequality gives
f) y ≤ 3x − 4
2 + (2)(3) < 10 or 8 < 10 which is clearly true and so
points on this side of the line will satisfy the inequality.
Solution This side of the line can now be shaded, as below.

Basics of Subsidiary Mathematics By Khalid M +256 754857296 khalidhkmsh@gmail.com 17


2.3. OPTIMIZATION 2. LINEAR PROGRAMMING

looking at. The place where all the rules work together is
called the ”feasible region.” Any point inside this region
follows all the rules.

2.3 Optimization

This requires maximizing and minimizing the objective


NOTE: Parts (c), (d),(e) and (f) are left as exercise.
function.
2.2.1 More Than One Inequality. For example
Example 1: A small store has two types (A and B) of
Find the region which satisfies the inequalities x ≤ 4, products, and they can stock a maximum of 10 products
y ≤ 2x , y + x ≥ 1 Write down the coordinates of the in total. They stock more of A type than B.
vertices of this region.
Describe this situation using inequalities and draw a graph
Solution. We start by ploting the line x = 4, shade the to show the feasible region in which these conditions are
region which is satisfied by the inequality x ≤ 4. met.
This is followed by adding the region which satisfies Solution
y ≤ 2x using a different type of shading, as shown. Fi-
Let
nally, add the region which is satisfied by y + x ≥ 1 using
a third type of shading x = number of product A stocked,

The result is as seen in the figure. The region which y = number of product B stocked,
has been shaded in all three different ways (the triangle A maximum of 10 products can be stocked so x + y ≤ 10
outlined in bold) satisfies all three inequalities.
stock more of A type than B then x > y

As neither x nor y can be negative, then x ≥ 0 and


y ≥ 0.

These inequalities are represented on the graph below

The coordinates of its vertices can be seen from the dia-


gram as(1, 2), (4, 5) and (4, 8).

NOTE: When you have lots of lines and need to shade


areas on a graph, it can get confusing. To make it clearer,
it’s better to shade outside the area unwanted region
and leave the area you want unshaded.

Imagine it like this: You’re using a ”shadow” to shade


the side of the line that doesn’t match the rule you’re Example 2:

Basics of Subsidiary Mathematics By Khalid M +256 754857296 khalidhkmsh@gmail.com 18


2.3. OPTIMIZATION 2. LINEAR PROGRAMMING

A manufacturer of printed circuits has a stock of 200 i.e. y = 3 and x = 6. So maximum profit occurs at the
resistors, 120 transistors, and 150 capacitors. The man- point (6, 3) and the profit is given by
ufacturer needs to produce two types of circuits:
P = 5(6) + 12(3) = Shs. 66
- Type A requires 20 resistors, 10 transistors, and 10 ca-
pacitors. - Type B requires 10 resistors, 20 transistors, This means that the manufacturer should produce 6 Type
and 30 capacitors. A circuits and 3 Type B circuits to maximize profit.
The profit on type A circuits is Shs.5, and the profit on
type B circuits is Shs. 12. Determine how many of each
circuit should be produced to maximize profit.

Solution

Variables: Let x be the number of Type A circuits pro-


duced, and y be the number of Type B circuits produced.

Constraints:

Resistor constraint: 20x + 10y ≤ 200


Transistor constraint: 10x + 20y ≤ 120
Capacitor constraint: 10x + 30y ≤ 150
Non-negativity constraint: x ≥ 0, y≥0 Example 3:

A farmer has 20 hectares of land available for growing


Objective Function: Maximize P = 5x + 12y tomatoes and pineapples. The farmer needs to decide
Solution: how much of each crop to grow. The cost per hectare
for tomatoes is Shs 30, and for pineapples, it’s Shs 20.
To solve this using the graphical method as shown
The farmer has a budget of Shs 480. Growing tomatoes
requires 1 man-day per hectare, while growing pineapples
requires 2 man-days per hectare. There are 36 man-days
available. The profit on tomatoes is Shs 100 per hectare,
and on pineapples, it’s Shs 120 per hectare. Find the
number of hectares of each crop the farmer should sow
to maximize profit.

Solution:

Variables: Let x be the number of hectares of toma-


toes to be grown, and y be the number of hectares of
pineapples to be grown.
Zooming in the feasible region we can see that as the
Constraints:
profit line moves to the right, the profit increases and so
the maximum profit corresponds to the last point touched
Budget Constraint: 30x + 20y ≤ 480
as the profit line moves out of the feasible region. This
Man-Days Constraint: x + 2y ≤ 36
is the point B, the intersection of 10x + 30y = 150 and
10x + 20y = 120 Solving these equations gives 10y = 30, Non-negativity Constraint: x ≥ 0, y≥0

Basics of Subsidiary Mathematics By Khalid M +256 754857296 khalidhkmsh@gmail.com 19


2.4. EXERCISE: 2. LINEAR PROGRAMMING

Objective Function: Maximize P = 100x + 120y hectares of tomatoes and 16 hectares of pineapples to
maximize profit.

2.4 Exercise:

1. A small firm builds two types of garden sheds. Type


A requires 2 hours of machine time and 5 hours of crafts-
man time. Type B requires 3 hours of machine time and
5 hours of craftsman time. Each day there are 30 hours of
machine time available and 60 hours of craftsman time.
The profit on each Type A shed is Ugx 60, and on each
We can see that C increases as the line (shown dotted)
Type B shed is Ugx 84. Determine The values need for
moves to the right. Continuing in this way, the maximum
the firm to obtain maximum profit.
profit will occur at the intersection of x + 2y = 36 and
x + y = 20 This means that the farmer should plant 4 CONTACT FOR COMPLETE BOOK

Basics of Subsidiary Mathematics By Khalid M +256 754857296 khalidhkmsh@gmail.com 20


3. MOVING AVERAGES, PRICE INDICES,
CORRELATION
3.1 Moving Averages Calculating moving averages helps us smooth out these
patterns to better understand the trend. We can use dif-
Moving averages is a method used to predict numbers in ferent numbers of data points, such as 2, 3, 4, 5, 6, or 7,
different situations, like business. It involves finding the to calculate moving averages. These averages are placed
average of changes over time. These time periods can in the middle of the data they come from.
range from seconds to months or years. 3.1.1 Graphs of Moving Averages. Many datasets ex-
We use moving averages to observe how things change hibit patterns influenced by various factors like seasons,
over time. For instance, we can track changes in pop- months, or even specific times of the day. When we rep-
ulation, rainfall, or growth. These changes create pat- resent this data on a line graph, it’s called a time series
terns called trends. Moving averages help us draw a line graph, and often, it shows erratic fluctuations.
through these patterns to make predictions about the fu- To make sense of such graphs, it’s crucial to separate
ture. these different seasons, each with its unique impact. One
Understanding the trend is important because it helps us method to achieve this is by using what we call ”moving
identify when things are different. For example, there averages.” These are designed to smooth out the large
might be specific times during the year when sales are ups and downs that can happen in data that changes over
lower than usual. time.

Here are some types of trends in data: When we plot moving averages on a graph, we can spot
cyclical trends in the data. By calculating the average
of two or more data points, we remove the sharp peaks
1. Secular Trend: When data consistently increases
and valleys. Consequently, the graph of moving averages
or decreases over a long period. An example is how
shows a more stable trend in the data’s changes.
high jump records keep improving or how winning
marathon times get faster. Where we place the moving average on the graph mat-
ters significantly. It should be marked at the middle point
2. Seasonal Variation: When data follows a pattern of the data we used for the calculation. We then con-
during specific months each year. Think of heating nect these points sequentially with straight lines, creating
bills, which are higher in colder months and lower what we call a ”trend line.” This trend line helps us pre-
in warmer ones. Umbrella sales go up in the rainy dict future values by extending it.
season and down when it’s sunny.
If the plotted points of moving averages don’t form a
straight line when connected, we simply draw the best-
3. Cyclical Variation: This occurs when periods of
fitting trend line through them.
prosperity are followed by downturns, and this cycle
repeats. Example: 3-points moving average Let’s consider a
set of data points listed below:
4. Random Variation: This happens when unpre-
dictable events like wars or stock market crashes
Data : x1 , x2 , x3 , x4 , x5 , x6 , . . . , xn
disrupt the trend. It’s hard to predict because these
events are irregular.

Basics of Subsidiary Mathematics By Khalid M +256 754857296 khalidhkmsh@gmail.com 21


3.1. MOVING AVERAGES 3. MOVING AVERAGES, PRICE INDICES, CORRELATION

To calculate the first 3-point moving average, we add the next three data points (x2 , x3 , x4 ).
the first three data points together, resulting in the first
To calculate the third 3-point moving average, we drop
3-point moving total: x1 + x2 + x3 . We then divide
the second data point x2 and include the fifth data point
this moving total by 3 to obtain the first 3-point moving
x5 . This results in the third 3-point moving average:
average: x1 +x32 +x3 . It’s important to place this moving x3 +x4 +x5
3
, positioned at the midpoint of the next three
average at the midpoint of the first three data points:
data points (x3 , x4 , x5 ).
(x1 , x2 , x3 ).
This process continues to obtain subsequent 3-point mov-
For the second 3-point moving average, we exclude the
ing averages, always placing the moving average at the
first data point x1 and include the next data point
midpoint of the data points.
x4 . This gives us the second 3-point moving average:
x2 +x3 +x4
, which should be positioned at the midpoint of Note: To keep things organized, it’s a good practice to
3
tabulate the moving averages in the format shown below:

Example The table below shows the tax collection of a town council in millions of shillings for six consecutive months.

(a). Construct the 3-month moving averages for the given data.

(b). Plot the 3-month moving averages and the original data on the same axes.

(c). Use your graphs to estimate the town council’s tax collection for the month of July.

Month Jan Feb Mar Apr May Jun


Tax(in Millions) 21.5 24.3 21.8 26.2 22.7 28.9

Solution

Year points 3- pts moving totals 3-pts moving average


Jan 21.5
Feb 24.3 67.6 22.53
Mar 21.8 72.3 24.10
Apr 26.2 70.7 23.57
May 22.7 77.8 25.93
x+51.6
Jun 28.9 x+51.6 3
July x

from the graph,


x + 51.6
= 27.05
3
x + 51.6 − 27.05 × 3
= 81.15 − 51.6
− 29.55
Therefore, the amount of tax is
29.55 × 1, 000, 000 = 29, 550, 000

Basics of Subsidiary Mathematics By Khalid M +256 754857296 khalidhkmsh@gmail.com 22


3.1. MOVING AVERAGES 3. MOVING AVERAGES, PRICE INDICES, CORRELATION

Basics of Subsidiary Mathematics By Khalid M +256 754857296 khalidhkmsh@gmail.com 23


3.1. MOVING AVERAGES 3. MOVING AVERAGES, PRICE INDICES, CORRELATION

Example: 4-points moving average To find the four-point moving averages, we do things the same way as before,
but this time, we add up four data points and then divide the total by four to get the four-point moving averages.
Remember, we still put the moving average in the middle of those data points.

The table below shows the number of bags of sugar sold by a certain wholesale shop from the year 2009 to 2012.
Year 1st Quarter 2nd Quarter 3rd Quarter 4th Quarter
2009 192 280 320 260
2010 300 360 380 270
2011 342 420 430 320
2012 424 480 510 412
(a). Calculate the four-point moving averages for the data.

(b). (i). On the same axes, plot the original data and the four-point moving averages.

(ii). Comment on the trend of the number of bags of sugar sold over the four-year period. (iii). Use your graph to
estimate the number of bags to be sold in the first quarter of 2013.

Sol:

Basics of Subsidiary Mathematics By Khalid M +256 754857296 khalidhkmsh@gmail.com 24


3.1. MOVING AVERAGES 3. MOVING AVERAGES, PRICE INDICES, CORRELATION

Year Quarter Bags of Sugar 4-Point Moving Total 4-Point Moving Average
2009 1st 192

2nd 280
1052 263
3rd 320
1160 290
4th 260
1240 310
2010 1st 300
1300 325
2nd 360
1310 327.5
3rd 380
1352 338
4th 270
1412 353
(a).
2011 1st 342
1462 365.5
2nd 420
1512 378
3rd 430
1594 398.5
4th 320
1654 413.5
2012 1st 424
1734 433.5
2nd 480
1826 456.5
3rd 510
1402+x
1402+x 4
4th 412

2013 1 x
from the graph,
1402 + x
= 467.5
4
1402 + x = 4 × 467.5
x = 1879 − 1402
= 468
The estimate the number of bags to be sold in the first quarter of 2013 is 468 bags.

Basics of Subsidiary Mathematics By Khalid M +256 754857296 khalidhkmsh@gmail.com 25


3.1. MOVING AVERAGES 3. MOVING AVERAGES, PRICE INDICES, CORRELATION

Figure 3.1: There is an increasing trend in the average number of bags of sugar over the four-year period.

Example: 5-points moving average The five-point moving averages are determined using the same process as
previously explained. In this case, we add up five consecutive data points and then divide the sum by five to calculate
the five-point moving averages. It’s essential to note that the moving average is positioned at the middle of these
data points.

Question: The provided table displays the monthly prices of a liter of milk throughout the year 2016.

(a). Utilize the data to compute the five-month moving totals and subsequently determine the five-month moving
averages.

(b). Create a single graph that illustrates both the given data and the moving averages on the same axes. Provide
insights into your findings.

(c). Utilize the graph from part (b) to estimate the milk price for:

(i). December 2015.

Basics of Subsidiary Mathematics By Khalid M +256 754857296 khalidhkmsh@gmail.com 26


3.1. MOVING AVERAGES 3. MOVING AVERAGES, PRICE INDICES, CORRELATION

(ii). January 2017.

Month Jan Feb Mar Apr May Jun Jul Aug Sep Oct Nov Dec
Price (shs) 1900 1905 1910 1910 1920 1930 1920 1945 1965 1960 1980 2000

Solution

Year points 5- pts moving totals 5-pts moving average


2015 Dec x
2016 Jan 1900
x+7625
Feb 1905 x + 7625 5
Mar 1910 9545 1909
Apr 1910 9575 1915
May 1920 9590 1918
Jun 1930 9625 1925
Jul 1920 9680 1936
Aug 1945 9720 1944
Sept 1965 9770 1954
Oct 1960 9850 1970
y+7905
Nov 1980 y+7905 5
Dec 2000
2017 Jan y

i) from the graph,


x + 7625
= 1900
5
x = 1875

ii) from the graph,


y + 7905
= 1978.5
5
y = 1987.5

Basics of Subsidiary Mathematics By Khalid M +256 754857296 khalidhkmsh@gmail.com 27


3.1. MOVING AVERAGES 3. MOVING AVERAGES, PRICE INDICES, CORRELATION

Figure 3.2: Comment: There is an increasing trend in the monthly prices of a litre of milk over the year 2016.

Example: 6-points moving average The table below shows the monthly sales of a Cars in (shs “000”) for the year

Basics of Subsidiary Mathematics By Khalid M +256 754857296 khalidhkmsh@gmail.com 28


3.1. MOVING AVERAGES 3. MOVING AVERAGES, PRICE INDICES, CORRELATION

2016.

(a). Calculate 6-point moving totals and hence the moving averages. [6]

(b). (i). Plot on the same axes actual sales and moving averages. Comment on the trend of sales during the year.

(ii). Determine the sales in January 2017.

Month Jan Feb Mar Apr May Jun Jul Aug Sep Oct Nov Dec
Sales 220 210 200 207 196 189 175 186 176 170 159 168

Solution: Table is left as an exercise to the learner. see graph in the figure

Basics of Subsidiary Mathematics By Khalid M +256 754857296 khalidhkmsh@gmail.com 29


3.1. MOVING AVERAGES 3. MOVING AVERAGES, PRICE INDICES, CORRELATION

Figure 3.3: Comment: There is a decreasing trend in the average sales during the year 2016.

3.1.2 Exercise.

Qtn 1. The number of Shirts sold by a shop for the first 10 weeks of the period February to April 2012 is as in the
table below.

Basics of Subsidiary Mathematics By Khalid M +256 754857296 khalidhkmsh@gmail.com 30


3.1. MOVING AVERAGES 3. MOVING AVERAGES, PRICE INDICES, CORRELATION

Week 1 2 3 4 5 6 7 8 9 10
Sales 7 4 6 4 5 3 4 1 5 2

(i) Draw a graph showing the weekly sales.


(ii) Calculate the 4 – weekly moving averages of sales.
(iii) On the same diagram as for part (i) draw the graph of the moving average. Which of these graphs gives
you a clear idea of the trend of sales and why.
(iv) Comment on the trend of sales for this period.

Qtn 2. The table shows the average monthly production of a certain commodity in thousands of tones.

Year 2000 2001 2002 2003 2004 2005 2006


Production 48 36 43 45 38 36 31

(a). On the same coordinate axes represent the average monthly production and the three-year moving averages
for the data.
(b). From the graph estimate the average monthly production in 2007.

Basics of Subsidiary Mathematics By Khalid M +256 754857296 khalidhkmsh@gmail.com 31


3.2. INDEX NUMBERS 3. MOVING AVERAGES, PRICE INDICES, CORRELATION

3.2 INDEX NUMBERS Solution


p2012
Price index = × 100
p2010
Index numbers are statistical tools used to represent 6300
= × 100
changes in a variable or a group of related variables over 4200
time, location, or other characteristics like income or job. = 150%
They show how much something has changed compared
So the price index or price relative is 150. This indicates
to a fixed past period.
that the price of a kilogram of sugar increased by 50%
Index numbers can be grouped based on what they mea- between 2010 and 2012.
sure. In business, index numbers are commonly used to
Example 2 One litre of petrol costs 2500 UGX in 2005
measure things like (i) price, (ii) quantity, (iii) value, and
and 3000 UGX in 2009. Taking 2005 as the base year,
so on. An example of an index number is a price index
find the price relative in 2009.
(also called a price relative).
Solution
Price Index (Price Relative) – The Price Index tells us p2009
Price relative = × 100
p2005
how the price of something changes over time. It mea-
3000
sures changes in prices of things between two periods. = × 100
2500
Terminology used in price index: = 120%

Generally, we compare current prices with prices in the This indicates that the price of petrol went up by 20%
past. The current time is called the ”current period,” between 2005 and 2009.
and some past time is called the ”base period.” These
3.2.2 Simple Aggregate Price Index. The simple ag-
periods could be days, weeks, months, years, or decades.
gregate price index is obtained by calculating the total
price of a group of items as a ratio of the total price of
ˆ Base Year: The year used to compare other prices the same group of items in the base year. It consists of
or the year on which price changes are based. expressing the aggregate price of all commodities in the
current year as a percentage of the aggregate price in the
ˆ Current Year: The year for which the index is cal-
base year.
culated or the year being compared with the base
P
year. p1
Simple aggregate price index, P = P × 100
p0
Sum of prices in current year

3.2.1 Simple Price Index. A simple price index mea- = × 100


Sum of prices in base year

sures how the price of one thing changes compared to the


P
base price. Where p1 is the sum of the prices for the current year
P
and p0 is the sum of the prices for the base year.
If the base period price is denoted as p0 and the cur-
rent period price is p1 , then the simple price index P is Example 1
calculated like this:
Below is a table showing the prices of items A, B, C, D,
E, and F in US dollars in 1995 and 2005. Using 1995
p1
P = × 100 as the base year, calculate the aggregate price index for
p0
these items.
Example 1 In January 2010, the price of a kilogram of
sugar was 4200 UGX. In January 2012, the price was 6300
UGX. Taking 2010 as the base year, find the price index.

Basics of Subsidiary Mathematics By Khalid M +256 754857296 khalidhkmsh@gmail.com 32


3.2. INDEX NUMBERS 3. MOVING AVERAGES, PRICE INDICES, CORRELATION

Item Price in 1995 ($) Price in 2005 ($) weighted contributions of different items to the overall
A 10 15 index and is calculated as:
B 20 25
C 15 30 P p1
D 25 35 p0
·w
Weighted Average Price Index = P × 100
E 30 40 w
F 40 45
Here, p1 and p0 represent the prices of individual items
Solution in the current year and base year, respectively. w repre-
sents the weight of each item, and Pw and P represent
To calculate the aggregate price index, we need to find
the aggregate prices in the current year and base year,
the sum of the prices for each year:
respectively.

The ”weight of items” mentioned here refers to how im-


Total price in 1995 = 10 + 20 + 15 + 25 + 30 + 40 = 140 portant each good or service is in terms of its contribution
to the overall consumption of households. In essence, it
Total price in 2005 = 15 + 25 + 30 + 35 + 40 + 45 = 190
reflects the relative significance of different items in the
total household spending.
Now, we can use these totals to calculate the aggregate
price index: 3.2.4 Weighted Aggregate Price Index. This is of-
ten referred to as a more accurate assessment of price
changes by considering the relative importance of differ-
Total price in 2005 ent items.
Aggregate price index, P = × 100
Total price in 1995 P P
190 p1 · w p1 · w 1
= × 100 Weighted Aggregate Price Index = P × 100 = P × 100
140 p0 · w p0 · w 0
≈ 135.71%
ExampleThe table below shows the price in shilling of
items A, B and C and their weights in 2010 and 2014.
So, the aggregate price index for these items is approxi-
mately 135.71. Item price 2010 price 2014 weight
3.2.3 A Weighted Aggregate Price Index. This is A shs.1500 shs.1800 4
also known as a Composite Index, is a statistical measure B shs.2500 shs.2800 6
used to represent changes in the overall price levels of a C shs.900 shs.800 5
group of items, where each item is given a specific weight
based on its relative importance. This index helps provide Taking 2010 as the base year, calculate;
a more accurate reflection of price changes over time. (a) simple aggregate price index for 2014.
This method of calculating a Composite Index is often (b) Price relative for each item for 2014
used to create a Weighted Average Price Index, also
(c) Find weighted price index for 2014.
known as a Cost of Living Index or Weighted Index. The
Weighted Average Price Index takes into account the (d) Find weighted aggregate price index for 2014.

Solution

Basics of Subsidiary Mathematics By Khalid M +256 754857296 khalidhkmsh@gmail.com 33


3.3. REGRESSION AND CORRELATION 3. MOVING AVERAGES, PRICE INDICES, CORRELATION

P2014 P2014
Item price 2010 price 2014 W P2010
× 100 P2010
W × 100 P2010 W P2014 W
A shs.1500 shs.1800 4 120.00 480.00 6000 7200
B shs.2500 shs.2800 6 112.00 672.00 15000 16800
C shs.900 shs.800 5 88.89 444.45 4500 4000
Total P
P2010 = 4900
P
P2014 = 5400
P
W = 15
P P2014
P2010
W × 100 = 1596.45
P
P2010 W = 22500
P
P2014 W = 24400

(a) simple aggregate price index for 2014. Item Term one Term two weight
P Soda shs.800 shs.1200 2
p1
Simple aggregate price index = P × 100 Chapati shs.1000 shs.1500 1
p0
5400 Samosa shs.900 shs.1200 3
= × 100
4900 Sweet shs.300 shs.400 4
= 110.2%
(a) Calculate the simple price index of each item in
term two,
(b) Price relative for each item for 2014
(b) Calculate Marion’s cost of living index in term
two,
A = 120.00% B = 112.00% C = 88.89%
(c) Calculate Marion’s simple aggregate index for
(c) Find weighted price index for 2014. term two taking term one as the 100% year.
P p1
p0
·W Qtn 2. The table below shows the expenditure of restau-
Weighted Price Index = P × 100
W rants for the years 2014 and 2016.
1596.45
=
15
= 106.43% Items Price 2014 (Shs) Price 2016 (Sh
Milk (per litre) 1000 1300
(d) Find weighted aggregate price index for 2014. Eggs (per tray) 6500 8300
P
p1 · w Sugar (per kg) 3000 3800
Weighted Aggregate Price Index = P × 100 Blue band 7000 9000
p0 · w
24400
= × 100 Taking 2014 as the base year, calculate for 2016
22500
= 109.804% the; (a) Price relative for each item.
(b) Simple aggregate Price index.
3.2.5 Exercise.
(c) Weighted aggregate Price index and comment
on your results.
(d) In 2016, the restaurant spent Shs.45, 000 on
buying these items. Using the index obtained in
Qtn1. The table below shows Marion’s expenditure on (c), find how much money the restaurant could
break time during School time in term have spent in 2014.

3.3 REGRESSION AND CORRE-


LATION

This section deals with examining the relationship be-


Basics of Subsidiary Mathematics By Khalid M +256 754857296tween
khalidhkmsh@gmail.com 34
two variables, where one variable is considered in-
3.4. SCATTER DIAGRAM/SCATTER GRAPH 3. MOVING AVERAGES, PRICE INDICES, CORRELATION

dependent while the other is dependent. This type of correlation, the Spearman’s value ranges are as follows:
relationship is referred to as ”bi-variates.”

3.3.1 Rank Correlation. Rank correlation is a method


used to assess the strength of the relationship between 0.0 − +0.19 : Very low positive correlation
two variables by ranking them. It involves assigning po- 0.2 − 0.39 : Low (weak) positive correlation
sitions to data points based on their values. 0.4 − 0.59 : Moderate positive correlation
Correlation is a measure of the connection between a pair 0.6 − 0.79 : High (substantial) positive correlation
of observations. Ranking involves arranging data points 0.8 − 0.99 : Very high positive correlation
in either ascending or descending order. +1 : Perfect positive correlation
There are two common methods for ranking data: Spear-
man’s and Kendall’s rank correlation. For a negative correlation, the Spearman’s value ranges
3.3.2 Spearman’s Rank Correlation. Spearman’s rank are as follows:
correlation coefficient, denoted as ρ, is used to calculate
rank correlation coefficients. It ranks data by assigning
positions to data points in ascending or descending or- 0.0 − −0.19 : Very low negative correlation
der. In cases where there are repeated values in the data, 0.2 − −0.39 : Low (weak) negative correlation
these values are given the same rank by averaging the 0.4 − −0.59 : Moderate negative correlation
positions they would have assumed.
0.6 − −0.79 : High (substantial) negative correlation
The formula for Spearman’s rank correlation 0.8 − −0.99 : Very high negative correlation
coefficient(ρ) is as follows:
−1 : Perfect negative correlation
6 d2
P
ρ=1−
n(n2 − 1) Note: We consider the size and sign of correlation while
excluding the work coefficient from our conclusion.
where: - n represents the number of pairs of observations
or pairs. - d is the difference between rankings. (b) Significance Levels: In cases where significance levels
are required (1% or 5% level of significance), the mag-
3.3.3 Drawing Conclusions. Drawing conclusions in-
nitude of the calculated value is compared to the values
volves interpreting the correlation coefficient obtained
in mathematical tables at those levels for a given n. If
from the calculation. There are two common ways to
the calculated value exceeds or matches the values in the
conclude: (a) Commenting on the Answer: For a positive
tables, it is considered significant; otherwise, it is not.

3.3.4 Examples. and an accurate scale on the graph for clarity.

Scatter plots help us visualize the apparent relationship


between two variables when plotted together. In this rep-
3.4 Scatter Diagram/Scatter resentation, the independent variable, denoted as X, is
Graph plotted on the horizontal axis, while the dependent vari-
able, Y , is plotted on the vertical axis.

A scatter diagram, also known as a scatter graph, is a 3.4.1 Types of Correlation. Correlation types explain
graphical representation of the relationship between two the relationship between the two plotted variables, de-
variables. It is essential to include a title, labeled axes,

Basics of Subsidiary Mathematics By Khalid M +256 754857296 khalidhkmsh@gmail.com 35


3.4. SCATTER DIAGRAM/SCATTER GRAPH 3. MOVING AVERAGES, PRICE INDICES, CORRELATION

noted as X and Y . These types include: negative, or no correlation, referring to the illustrations
above.
(a) Negative Correlation: In this case, an increase in the
independent variable (X) results in a decrease in the de- 3.4.2 Drawing a Line of Best Fit. The line of best
pendent variable (Y ). fit is drawn to ensure it passes through the mean point
(M0 (x, y)) of the given data. Two formats are generally
Dependent (Y ) used:

1. Line of Best Fit of x on y: - Plot the mean (M0 (x, y))
↓↓↓↓↓↓↓↓↓↓↓↓↓↓↓↓
on the scatter. - Draw a line parallel to the x-axis (using
Independent (X)
the imaginary eye idea). - Find the mean (Ma ) of the
points above. - Find the mean (Mb ) of the points below.
(b) Positive Correlation: Here, an increase in the indepen-
- Draw a solid line passing through M0 , Ma , and Mb ,
dent variable (X) leads to an increase in the dependent
forming the line of best fit. This line is preferred when y
variable (Y ).
values are more accurate.
Dependent (Y ) 2. Line of Best Fit of y on x: - Plot the mean (M0 (x, y))
↑ on the scatter. - Draw a line parallel to the y-axis (using
↑↑↑↑↑↑↑↑↑↑↑↑↑↑↑↑ the imaginary eye idea). - Find the mean (Ml ) of the
Independent (X) points on the left. - Find the mean (Mr ) of the points
on the right. - Draw a solid line passing through M0 , Ml ,
(c) Zero/No/Neutral Correlation: In this scenario, and Mr , forming the line of best fit. This line is preferred
changes in the independent variable (X) have little to when x values are more accurate.
no effect on the dependent variable (Y ), and vice versa.
For increased accuracy, ensure that the line of best fit
Dependent (Y ) passes through the mean of all points.
↑ 3.4.3 Equation of the Line of Best Fit. The equation
↓↓↓↓↓↓↓↓↓↓↓↓↓↓↓↓ of the line of best fit is determined by selecting any two
Independent (X) points from the drawn line and using them to calculate
the equation of the line in the form y = mx + c using
To comment on the graph, observe the trend of the the analytical approach.
scattered points and classify the relationship as positive,

3.4.4 Example. The table below displays the height and weight of 8 students in a certain school:

CONTACT FOR COMPLETE BOOK

Basics of Subsidiary Mathematics By Khalid M +256 754857296 khalidhkmsh@gmail.com 36


4. VECTORS AND TRIGONOMETRY
4.1 Vectors

In mathematics and physics, vectors are quantities that have both magnitude and direction. They are often represented
using various notations.

4.1.1 Vector Notation. A vector is typically represented as v and can be denoted in different forms:
" #
x
ˆ Column Notation: v =
y
h i
ˆ Row Notation: v = x, y

ˆ Angle-Brackets Notation: v = hx, yi

4.1.2 Displacement Vector. A displacement vector, denoted as d, represents the change in position from one point
to another.

4.1.3 Position Vector. A position vector, denoted as r, locates a point with respect to a reference point.

4.2 Vector Operations

4.2.1 Addition and Subtraction of Vectors. Vectors can be added or subtracted component-wise. For vectors
u = hu1 , u2 i and v = hv1 , v2 i:
u + v = hu1 + v1 , u2 + v2 i
u − v = hu1 − v1 , u2 − v2 i

4.2.2 Multiplication by a Scalar. A vector can be multiplied by a scalar, resulting in a vector with the same
direction (if the scalar is positive) or the opposite direction (if the scalar is negative).

4.2.3 Magnitude and Direction of a Vector. The magnitude of a vector v = hx, yi is given by:
p
|v| = x2 + y 2

The direction of v is given by the angle θ it makes with the positive x-axis:

y
tan θ =
x

4.2.4 Parallel Vectors. Two vectors are parallel if they have the same or opposite directions.

4.2.5 Equal Vectors. Two vectors are equal if they have the same magnitude and direction.

Basics of Subsidiary Mathematics By Khalid M +256 754857296 khalidhkmsh@gmail.com 37


4.3. EXAMPLES 4. VECTORS AND TRIGONOMETRY

4.2.6 Dot/Scalar Product of Two Vectors. The dot product of vectors u and v is defined as:

u · v = u1 v1 + u2 v2

4.2.7 Angle Between Vectors. The angle θ between two vectors u and v can be found using the dot product:

u·v
cos θ =
|u| · |v|

4.2.8 Perpendicular Vectors. Two vectors are perpendicular (orthogonal) if their dot product is zero.

4.3 Examples

Example 1

Given vectors a = h2, −3i and b = h−1, 4i, find a + b and a − b.

Example 2

Find the magnitude and direction of the vector v = h3, 4i. Example 3 Given vectors u = h3, 1i and v = h−2, 4i,
find the angle between them.

Solution: To find the angle θ between vectors u and v, we can use the dot product formula:

u·v
cos θ =
|u| · |v|

First, calculate the dot product:


u · v = (3)(−2) + (1)(4)
= −6 + 4
= −2
Now, calculate the magnitudes of u and v:
√ √ √
|u| = 32 + 12 = 9 + 1 = 10
p √ √
|v| = (−2)2 + 42 = 4 + 16 = 20

Now, substitute these values into the formula for cos θ:

−2 −2 −2 −1
cos θ = √ √ =√ = √ = √
( 10)( 20) 200 10 2 5 2

Taking the inverse cosine, we find the angle θ:


 
−1 −1
θ = cos √
5 2

Use a calculator to approximate θ in degrees and radians.

Basics of Subsidiary Mathematics By Khalid M +256 754857296 khalidhkmsh@gmail.com 38


4.4. TRIAL EXERCISE 4. VECTORS AND TRIGONOMETRY

Example 4

Determine if the vectors a = h2, −3i and b = h3, 2i are perpendicular.

Solution: Two vectors are perpendicular if their dot product is zero. Calculate the dot product of a and b:

a · b = (2)(3) + (−3)(2)
=6−6
=0

Since a · b = 0, vectors a and b are perpendicular.

4.4 Trial Exercise

Evaluate the following:

1. u = h1, −2i + h−3, 5i

2. v = 2h2, 1i − 3h−1, 2i

3. Find the dot product of x = h3, −1i and y = h−2, 4i.

4. Determine whether vectors p = h1, −3i and q = h−2, 6i are parallel, equal, or neither.

4.5 Trigonometry

4.5.1 Trigonometric Ratios. Trigonometric ratios are relationships between the sides of a right-angled triangle.
There are three primary trigonometric ratios: sine (sin), cosine (cos), and tangent (tan).

4.5.2 Expressions of the Six Trigonometric Ratios. For a right-angled triangle with angle θ:

opposite adjacent opposite


sin θ = , cos θ = , tan θ =
hypotenuse hypotenuse adjacent

The reciprocals of these ratios are: cosecant (csc), secant (sec), and cotangent (cot):

1 1 1
csc θ = , sec θ = , cot θ =
sin θ cos θ tan θ

4.5.3 Special Angles. Common special angles include: 30◦ , 45◦ , and 60◦ .

4.5.4 Graphs of Trigonometric Functions. The graphs of sin θ and cos θ are periodic, oscillating between -1 and
1.

Basics of Subsidiary Mathematics By Khalid M +256 754857296 khalidhkmsh@gmail.com 39


4.5. TRIGONOMETRY 4. VECTORS AND TRIGONOMETRY

Trigonometric Identities

4.5.5 Basic Trigonometric Identities.


sin2 θ + cos2 θ = 1

tan2 θ + 1 = sec2 θ

cot2 θ + 1 = csc2 θ

Quadratic Equations

4.5.6 Introduction to Quadratic Equations. A quadratic equation is a polynomial equation of the second degree:

ax2 + bx + c = 0

Basics of Subsidiary Mathematics By Khalid M +256 754857296 khalidhkmsh@gmail.com 40


5. DIFFERENTIATION, INTEGRATION AND
CURVE SKETCHING
5.1 Derivatives

In calculus, differentiation is the process of finding the rate of change of a function. It provides us with tools to
calculate derivatives for various types of functions. In this section, we’ll explore derivatives of linear, quadratic, and
cubic functions and discuss the first and second derivatives for functions with integer exponents.

5.1.1 Derivatives of Linear Functions. A linear function f (x) = ax+b has a constant rate of change. Its derivative
f 0 (x) is equal to the coefficient of x:

d
f (x) = ax + b ⇒ f (x) = a
dx

Example

Find the derivative of f (x) = 3x + 2:

d
(3x + 2) = 3
dx

5.1.2 Derivatives of Quadratic Functions. A quadratic function f (x) = ax2 + bx + c has a parabolic shape. Its
derivative f 0 (x) can be found using the power rule:

d
f (x) = ax2 + bx + c ⇒ f (x) = 2ax + b
dx

Example

Find the derivative of f (x) = 2x2 + 3x − 1:

d
(2x2 + 3x − 1) = 4x + 3
dx

5.1.3 Derivatives of Cubic Functions. A cubic function f (x) = ax3 + bx2 + cx + d has a more complex shape.
Its derivative f 0 (x) can also be found using the power rule:

d
f (x) = ax3 + bx2 + cx + d ⇒ f (x) = 3ax2 + 2bx + c
dx

Example

Find the derivative of f (x) = 3x3 − 2x2 + 5x − 1:

Basics of Subsidiary Mathematics By Khalid M +256 754857296 khalidhkmsh@gmail.com 41


5.1. DERIVATIVES 5. DIFFERENTIATION, INTEGRATION AND CURVE SKETCHING

d
(3x3 − 2x2 + 5x − 1) = 9x2 − 4x + 5
dx

5.1.4 First Derivative for Integer Exponents. When dealing with functions of the form f (x) = xn , where n is a
positive integer, the first derivative f 0 (x) can be calculated using the power rule:

d
f (x) = xn ⇒ f (x) = nxn−1
dx

This rule applies to functions with integer exponents n where n ≥ 1.

Example

Find the derivative of f (x) = x3 :

d 3
(x ) = 3x2
dx

5.1.5 Second Derivative for Integer Exponents. To find the second derivative f 00 (x) for functions f (x) = xn ,
where n is a positive integer, you can apply the power rule twice:

n d d2
f (x) = x ⇒ f (x) = nxn−1 ⇒ 2
f (x) = n(n − 1)xn−2
dx dx

This rule applies to functions with integer exponents n where n ≥ 2.

Example

Find the second derivative of f (x) = x4 :

d2 4
(x ) = 12x2
dx2

5.1.6 Natural Logarithm in Differentiation. The derivative of the natural logarithm function f (x) = ln(x) is
given by:

d 1
f (x) = ln(x) ⇒ f (x) =
dx x

This result is essential in various applications, particularly in solving exponential growth and decay problems.

Example

Find the derivative of f (x) = ln(x):

d 1
(ln(x)) =
dx x

Basics of Subsidiary Mathematics By Khalid M +256 754857296 khalidhkmsh@gmail.com 42


5.2. CRITICAL POINTS AND THE FIRST DERIVATIVE
5. DIFFERENTIATION, INTEGRATION AND CURVE SKETCHING

5.2 Critical Points and the First Derivative

In calculus, critical points of a function are values of x where the first derivative f 0 (x) equals zero or is undefined.
Critical points play a crucial role in determining whether a function has a local minimum, local maximum, or neither.

5.2.1 Finding Critical Points. To find critical points, follow these steps:

1. Compute the first derivative f 0 (x) of the given function f (x). 2. Set f 0 (x) = 0 and solve for x. 3. The solutions
to f 0 (x) = 0 are the critical points of the function.

Example

Find the critical points of the function f (x) = x3 − 6x2 + 9x + 4.

1. Compute f 0 (x):
f 0 (x) = 3x2 − 12x + 9

2. Set f 0 (x) = 0 and solve for x:


3x2 − 12x + 9 = 0

Using the quadratic formula, we find two critical points: x = 1 and x = 3.

5.2.2 Determining Maximum and Minimum. After finding the critical points, you can determine whether each
critical point represents a local maximum, a local minimum, or neither:

- If f 0 (x) > 0 to the left of a critical point and f 0 (x) < 0 to the right of that point, it’s a local maximum. - If
f 0 (x) < 0 to the left of a critical point and f 0 (x) > 0 to the right of that point, it’s a local minimum. - If f 0 (x) does
not change sign at a critical point, it’s neither a maximum nor a minimum.

Example

Determine the nature of the critical points of the function f (x) = x3 − 6x2 + 9x + 4.

At x = 1: - To the left of x = 1, f 0 (x) = 3(1)2 −12(1)+9 = 0. - To the right of x = 1, f 0 (x) = 3(2)2 −12(2)+9 = −9.
Since f 0 (x) changes sign from positive to negative, x = 1 is a local maximum.

At x = 3: - To the left of x = 3, f 0 (x) = 3(2)2 −12(2)+9 = −9. - To the right of x = 3, f 0 (x) = 3(3)2 −12(3)+9 = 0.
Since f 0 (x) changes sign from negative to positive, x = 3 is a local minimum.

5.3 Formulation of Functions

In calculus, you can use functions to model and solve real-life problems. Two common types of real-life functions
are profit functions and cost reduction functions. These functions help businesses make decisions about pricing and
production.

5.3.1 Profit Functions. A profit function models the profit (revenue minus cost) of a business as a function of a
variable, such as the number of units sold. To maximize profit, find the critical point where the derivative of the
profit function is zero, and determine whether it represents a maximum.

Basics of Subsidiary Mathematics By Khalid M +256 754857296 khalidhkmsh@gmail.com 43


5.3. FORMULATION OF FUNCTIONS 5. DIFFERENTIATION, INTEGRATION AND CURVE SKETCHING

Example

A company’s profit function is given by P (x) = −x2 + 40x − 100, where x is the number of units sold. Find the
production quantity that maximizes profit.

1. Compute P 0 (x):
P 0 (x) = −2x + 40

2. Set P 0 (x) = 0 and solve for x:

−2x + 40 = 0 =⇒ 2x = 40 =⇒ x = 20

The critical point is x = 20. To determine whether it’s a maximum, check the sign change of P 0 (x) around this
point.

- To the left of x = 20, P 0 (x) = −2(19) + 40 = 2 > 0. - To the right of x = 20, P 0 (x) = −2(21) + 40 = −2 < 0.

Since P 0 (x) changes sign from positive to negative, x = 20 represents a local maximum.

5.3.2 Cost Reduction Functions. A cost reduction function models the cost of production as a function of a
variable, such as the quantity produced. To minimize costs, find the critical point where the derivative of the cost
reduction function is zero, and determine whether it represents a minimum.

Example

A company’s cost reduction function is given by C(x) = x2 − 20x + 150, where x is the quantity produced. Find the
production quantity that minimizes costs.

1. Compute C 0 (x):
C 0 (x) = 2x − 20

2. Set C 0 (x) = 0 and solve for x:

2x − 20 = 0 =⇒ 2x = 20 =⇒ x = 10

The critical point is x = 10. To determine whether it’s a minimum, check the sign change of C 0 (x) around this point.

- To the left of x = 10, C 0 (x) = 2(9) − 20 = −2 < 0. - To the right of x = 10, C 0 (x) = 2(11) − 20 = 2 > 0.

Since C 0 (x) changes sign from negative to positive, x = 10 represents a local minimum.

5.3.3 Real-Life Applications. Calculus is widely used in real-life situations to optimize functions. Examples include:

- Maximizing profit by determining the optimal price and production quantity. - Minimizing cost by optimizing
production processes. - Maximizing revenue in marketing campaigns. - Minimizing travel time in route planning.

By formulating functions and finding critical points, you can make informed decisions in various fields.

Basics of Subsidiary Mathematics By Khalid M +256 754857296 khalidhkmsh@gmail.com 44


5.4. SKETCHING QUADRATIC CURVES 5. DIFFERENTIATION, INTEGRATION AND CURVE SKETCHING

5.4 Sketching Quadratic Curves

5.4.1 Turning points. In calculus, turning points of a quadratic curve are points where the curve changes direction,
transitioning from increasing to decreasing (local maximum) or from decreasing to increasing (local minimum).
Calculating turning points involves finding the coordinates (x, y) where the derivative is zero.

Gradient Method To find turning points using the gradient method:

1. Compute the first derivative f 0 (x) of the quadratic function f (x). 2. Set f 0 (x) = 0 and solve for x. 3. Substitute
these values of x into the original function to find the corresponding y-coordinates.

5.4.2 Second Derivative Method. Alternatively, you can use the second derivative to determine the nature of
turning points:

1. Compute the first and second derivatives, f 0 (x) and f 00 (x), of the quadratic function f (x). 2. Find the values of
x where f 0 (x) = 0. These are potential turning points. 3. Examine the sign of f 00 (x) at each potential turning point:
- If f 00 (x) > 0, it’s a local minimum point. - If f 00 (x) < 0, it’s a local maximum point. - If f 00 (x) = 0, the nature of
the turning point is unclear.

5.5 Determining the Nature of Turning Points

Once you’ve found potential turning points, you can determine their nature:

- For a turning point at x = a: - If f 00 (a) > 0, it’s a local minimum point. - If f 00 (a) < 0, it’s a local maximum point.

Example Consider the quadratic function f (x) = 2x2 − 8x + 6.

Step 1: Find the First and Second Derivatives

The first derivative f 0 (x) is given by:


f 0 (x) = 4x − 8

The second derivative f 00 (x) is:


f 00 (x) = 4

Step 2: Find Potential Turning Points

To find potential turning points, set f 0 (x) = 0 and solve for x:

4x − 8 = 0

Solving for x:
4x = 8 =⇒ x = 2

So, the potential turning point is at x = 2.

Step 3: Determine the Nature of the Turning Point

Basics of Subsidiary Mathematics By Khalid M +256 754857296 khalidhkmsh@gmail.com 45


5.6. INTERCEPTS 5. DIFFERENTIATION, INTEGRATION AND CURVE SKETCHING

To determine the nature of the turning point at x = 2, use the second derivative f 00 (x):

Since f 00 (x) is a constant (4) and it’s greater than zero (f 00 (x) > 0), this means that the turning point at x = 2 is a
local minimum point.

So, for the quadratic function f (x) = 2x2 − 8x + 6, the turning point at x = 2 is a local minimum point.

You can also find the corresponding y-coordinate by substituting x = 2 back into the original function:

f (2) = 2(2)2 − 8(2) + 6 = 4 − 16 + 6 = −6

So, the turning point is (2, −6), and it represents a local minimum point on the curve.

5.6 Intercepts

Intercepts are points where a curve intersects the coordinate axes. For a quadratic function f (x), you can find the
following intercepts:

- Y-Intercept (y-intercept): To find the y-intercept, set x = 0 in the quadratic function and solve for y.

- X-Intercepts (x-intercepts or roots): To find the x-intercepts, set y = 0 in the quadratic function and solve for
x. These are also the solutions to the quadratic equation.

5.7 Curve Sketching

Curve sketching involves creating a visual representation of the quadratic curve using the information you’ve gathered,
including turning points, intercepts, and the shape of the curve.

5.7.1 Steps for Curve Sketching. 1. Find the y-intercept and x-intercepts. 2. Determine the turning points and
their nature (maximum or minimum). 3. Consider the leading coefficient of the quadratic function: - If it’s positive,
the parabola opens upward. - If it’s negative, the parabola opens downward. 4. Sketch the curve accordingly, ensuring
it passes through the intercepts and has the appropriate concavity (upward or downward).

5.8 Integration

5.9 Indefinite Integrals

In calculus, indefinite integrals are used to find antiderivatives of functions. An antiderivative is a function whose
derivative is equal to the given function. The notation for an indefinite integral is:

Z
f (x) dx

Basics of Subsidiary Mathematics By Khalid M +256 754857296 khalidhkmsh@gmail.com 46


5.10. DEFINITE INTEGRALS 5. DIFFERENTIATION, INTEGRATION AND CURVE SKETCHING

Here, f (x) is the function to be integrated, and dx indicates the variable of integration.

5.9.1 Constant of Integration. When we find an antiderivative, there is an arbitrary constant introduced, denoted
as C, called the constant of integration. This constant accounts for all possible antiderivatives of the same function.
The general form of the indefinite integral is:

Z
f (x) dx = F (x) + C

Here, F (x) is an antiderivative of f (x), and C is the constant of integration.

Example

Find the indefinite integral of f (x) = 3x2 :

Z
3x2 dx = x3 + C

5.10 Definite Integrals

Definite integrals are used to calculate the net area under a curve between two specific points on the x-axis. The
notation for a definite integral is:

Z b
f (x) dx
a

Here, f (x) is the function to be integrated, and [a, b] are the limits of integration.

5.10.1 Evaluating Definite Integrals. To evaluate definite integrals, we find the antiderivative of the function and
then apply the Fundamental Theorem of Calculus:

Z b
f (x) dx = F (b) − F (a)
a

Where F (x) is an antiderivative of f (x).

Example

Evaluate the definite integral:

Z 3
(2x − 1) dx
1

Z 3
(2x − 1) dx = [x2 − x]31 = (32 − 3) − (12 − 1) = 9 − 3 − 1 + 1 = 6
1

Basics of Subsidiary Mathematics By Khalid M +256 754857296 khalidhkmsh@gmail.com 47


5.11. NATURAL LOGARITHM IN INTEGRATION
5. DIFFERENTIATION, INTEGRATION AND CURVE SKETCHING

5.11 Natural Logarithm in Integration

The natural logarithm, denoted as ln(x), frequently appears in integration, especially when dealing with functions
involving exponentials. The integral of x1 with respect to x is the natural logarithm:

Z
1
dx = ln |x| + C
x

Here, C is the constant of integration, and |x| denotes the absolute value of x.

Example

Find the integral of x1 :

Z
1
dx = ln |x| + C
x

Basics of Subsidiary Mathematics By Khalid M +256 754857296 khalidhkmsh@gmail.com 48


6. Probability Theory
Probability is the likelihood of an event occurring. This Where: n(A) is the number of elements in event A, and
concept originated from games of chance, such as coin n(S) is the number of elements in the sample space S.
tossing, dice rolling, and even predicting the gender of a
6.0.5 Properties of Probabilities.
newborn.

6.0.1 Key Terms in Probability Theory.


ˆ P (A) ≥ 0: Probabilities are non-negative.
6.0.2 Sample Space (S). The sample space refers to
the set of all possible outcomes of an experiment. Each
ˆ P (A) ≤ 1: Probabilities do not exceed 1.
individual outcome within the sample space is known as
a sample point.
ˆ P (S) = 1: The sum of all probabilities within a
For example: sample space is equal to 1.

ˆ Rolling a six-sided die: Sample Space (S) = {1, 2,


3, 4, 5, 6}, and the sample points are 1, 2, 3, 4, 5, 6.0.6 Intersection of Events. If A and B are two
6. events, their intersection, denoted as A ∩ B, consists of
outcomes that belong to both A and B. The probability
ˆ Rolling a four-sided tetrahedron: Sample Space (S)
of both A and B occurring is represented as P (A ∩ B).
= {1, 2, 3, 4}, and the sample points are 1, 2, 3,
4. 6.0.7 Union of Events. If A and B are two events,
their union, denoted as A ∪ B, consists of outcomes that
Sample spaces can be generated through methods such belong to either A or B or both. The probability of either
as tables of outcomes, tree diagrams, permutations, and A or B occurring is represented as P (A ∪ B).
combinations.
For disjoint events, the probability of their union can be
6.0.3 Event. An event is a subset of a sample space, calculated as:
representing specific outcomes or combinations of out-
comes. P (A ∪ B) = P (A) + P (B) − P (A ∩ B)
For example:
Additionally, the probability that either A or B (but not
ˆ Tossing a six-sided die: If the desired event is to ob- both) occurs is given by:
tain an ’odd’ number, then the event E is defined
as E = {1, 3, 5}. P (A ∪ B but not both) = P (A) + P (B) − 2P (A ∩ B)

ˆ Tossing a fair coin three times: If the event of inter-


est is obtaining at least two heads, then the event 6.0.8 Complement of Events. If A is an event within
E is E = {HHH, HHT, HT H, T HH}. a sample space S, the complement of A, denoted as Ac ,
consists of all sample points not in A. Sometimes, we
6.0.4 Probability Function. The probability function use ”not A” to represent the complement.
of an event A, denoted as P (A), is the sum of the prob-
6.0.9 Mutually Exclusive Events (Disjoint Events).
abilities of all sample points in A. Mathematically, it’s
Two events A and B are mutually exclusive when their
expressed as:
n(A) intersection is empty, meaning they share no common
P (A) = sample points (A ∩ B = ∅). In this case, P (A ∩ B) = 0.
n(S)

Basics of Subsidiary Mathematics By Khalid M +256 754857296 khalidhkmsh@gmail.com 49


6. PROBABILITY THEORY

6.0.10 Independence of Events. Two events are in- Problem: One letter is selected at random from the word
dependent when the occurrence of one event does not ”SUBSIDIARYMATHEMATICS.” Find the probability of
influence the probability of the other event. In other selecting
words, they are unrelated, and the outcome of one event
does not affect the outcome of the second event. For 1. An ”A”
example, throwing a die and flipping a coin are inde- 2. An ”E”
pendent events. For two independent events A and B,
P (A ∩ B) = P (A) × P (B). The word ”SUBSIDIARYMATHEMATICS” has a sample
space of 21 letters.
Examples
number of A’s 3 1
Problem: The numbers 1 to 20 are each written on a 1. P (selecting A) = total number of letters
= 21
= 7
card. The 20 cards are mixed together. One card is cho- number of E’s 1
2. P (selecting E) = total number of letters
= 21
sen at random from the pack. Find the probability that
the number on the card is: Problem

Let’s consider another example:


1. Even
Problem: The two sides of a coin are known as ”head”
2. A factor of 24 and ”tail.” Two coins are tossed at the same time. Illus-
trate the possible outcomes on a sample space diagram
3. Prime
and find the probability of obtaining

Solution: 1. Two heads

Let S be the sample space such that S = 2. A head and a tail.


{1, 2, 3, 4, 5, 6, 7, 8, . . . , 20}.
The sample space S for the outcomes when the two coins
number of even numbers 10 1 are tossed is as shown:
1. P (Even) = total number of numbers in the pack
= 20
= 2

number of factors of 24
2. P (a factor of 24) = total number of numbers in the pack HH HT
The factors of 24 are: F24 = {1, 2, 3, 4, 6, 8, 12, 24}. TH TT

number of factors of 24 = |F24 | = 8 Let A be the event ”two heads are obtained.”
8 2
Therefore, P (a factor of 24) = 20
= 5 A = {HH} =⇒ |A| = 1

3. Prime numbers in the pack = {2, 3, 5, 7, 11, 13, 17, 19} Therefore, P (A) = |A|
|S|
= 14 . The probability that two
P (Prime) = number of prime numbers in the pack heads are obtained is 41 .
total number of numbers in the pack

P (Prime) = 8
= 2 Let E be the event ”a head and a tail is obtained.”
20 5

E = {HT, TH} =⇒ |E| = 2


So, the probabilities are: |E| 2
Therefore, P (E) = |S|
= 4
= 21 .
1
(a) P (Even) = 2 Problem: A fair coin is tossed, and a fair die is rolled.
2
(b) P (a factor of 24) = 5
Find the probability of obtaining a ’head’ and a ’six.’
2
(c) P (Prime) = 5
Solution:

Basics of Subsidiary Mathematics By Khalid M +256 754857296 khalidhkmsh@gmail.com 50


6. PROBABILITY THEORY

When a coin is tossed once, the sample space is: S = two numbers on which the dice land. Find the possibility
{H, T } where H denotes a ’head’ and T a ’tail.’ So space and the probability of each element of the space.
P (H) = P (T ) = 12 .
9. One ball is selected at random from a bag containing
Similarly, the sample space when a die is tossed once is: 12 balls, of which x are white. (a) What is the probabil-
S = {1, 2, 3, 4, 5, 6}. So P (six) = 16 . ity of selecting a white ball? (b) When a further 6 white
balls are added, the probability of selecting a white ball
The two events are independent. Therefore,
is doubled. Find x.
P (head and six) = P (H) × P (six) = 12 × 1
6
1
= 12 .
10. A bead is drawn from a container containing 10 red,
6.0.11 Exercise:. 1. An ordinary die is thrown. Find the
15 black, 5 green, and 10 yellow beads. Find the proba-
probability that the number obtained (a) is a multiple of
bility that the bead is (a) black, (b) not green, (c) red or
3, (b) is less than 7, (c) is a factor of 6.
black, (d) not blue.
2. A bag contains 6 red balls and 4 green balls. (a) Find
6.0.12 Probability and Set Theory. There are sev-
the probability of selecting at random (i) a red ball, (ii)
eral fundamental laws and notations that relate to set
a green ball. (b) One red ball is removed from the bag.
theory. These laws govern how probabilities are assigned
Find the new probability of selecting at random: (i) a red
to events within a sample space. Here are some of the
ball, (ii) a green ball.
basic laws of probability in relation to set theory:
3. From a set of cards numbered 1 to 20, a card is drawn
Probabilities can be illustrated on a Venn diagram. The
at random. Find the probability that the number (a) is
rectangle represents the entire sample space, and the cir-
divisible by 4, (b) is greater than 15, (c) is divisible by 4
cle represents the event A, as shown below.
and greater than 15.

4. One letter is selected at random from the word ’UN-


NECESSARY.’ Find the probability of selecting: (a) an
R, (b) an E, (c) an O.

5. Two ordinary dice are thrown. Find the probability


that (a) the sum on the two dice is 3, (b) the sum on
the two dice exceeds 9, (c) the two dice show the same
number. Combinations of probabilities can be shown on a Venn
6. An ordinary die and a fair coin are thrown together. diagram, as follows;
Show the possible outcomes on a sample space diagram (i) A and B, A ∩ B. This is the overlap of the regions
and find the probability that (a) a head and a 2 are ob- corresponding to A and B as shown by the shaded region
tained, (b) a tail and a 7 are obtained, (c) a head and an in the figure below.
even number are obtained.

7. The letters of the word ’INDEPENDENCE’ are written


on individual cards, and the cards are put into a box. A
card is selected at random from the box. Find the prob-
ability of obtaining: (a) a card with the letter E, (b) a
card with the letter N.

8. Two tetrahedral dice (4-sided), each with faces labeled


(ii) A ∪ B is the region of points in either the A region
1, 2, 3, and 4, are thrown. The score is the sum of the
or the B region (or both). Note that the word or is in-

Basics of Subsidiary Mathematics By Khalid M +256 754857296 khalidhkmsh@gmail.com 51


6. PROBABILITY THEORY

clusive. ”A or B” means ”A or B or both”, as shown by 4. Law of Total Probability: For two events A and
the shaded region below. B where A and A0 , B and B 0 are complementary: (i)
P (B) = P (A ∩ B) + P (A0 ∩ B)

(ii) P (A) = P (A ∩ B) + P (A ∩ B 0 )

(iii) Ā is the region of points that are not in A, as shown


by the shaded region below.

(iii) P (A0 ) = P (A0 ∩ B) + P (A0 ∩ B 0 )

(iv) P (B 0 ) = P (A ∩ B 0 ) + P (A0 ∩ B 0 )
6.0.13 Basic Probability Laws. The laws are:

1. Addition Law for Non-mutually Exclusive


Events: For any two events A and B, the probability
of their union is given by:

5. De Morgan’s Laws: For any two events A and B:


P (A ∪ B) = P (A) + P (B) − P (A ∩ B)
(i) P (A ∩ B) = P (A0 ) ∪ P (B 0 )
2. Law of Total Probability: If B1 , B2 , . . . , Bn form (ii) P (A ∪ B) = P (A0 ) ∩ P (B 0 )
a partition of the sample space S, then for any event A,
6. Multiplicative Law for Independent Events (And
the probability of A can be expressed as:
Rule): If events A and B are independent:
n
X
P (A) = P (A ∩ Bi ) P (A ∩ B) = P (A) · P (B)
i=1

2. Complementary Law:
6.0.14 Contingency Table. A contingency table pro-
vides a way of portraying data that can facilitate calcu-
P (A) + P (A0 ) = 1 =⇒ P (A0 ) = 1 − P (A)
lating probabilities. We use a contingency table to rep-
resent the probabilities of two events, A and B, which
3. Additive Law: For any two events A and B: may or may not be independent. The contingency table
might look like this:
P (A ∪ B) = P (A) + P (B) − P (A ∩ B)
Event B B0
If A and B are mutually exclusive, P (A ∩ B) = 0, and A P (A ∩ B) P (A ∩ B 0 )
thus, P (A ∪ B) = P (A) + P (B). A0 P (A0 ∩ B) P (A0 ∩ B 0 )

Basics of Subsidiary Mathematics By Khalid M +256 754857296 khalidhkmsh@gmail.com 52


6. PROBABILITY THEORY

Some important relationships are visible in the contin- (b) From P (A ∪ B) = P (A) + P (B) − P (A ∩ B):
gency table: 1. P (A) = P (A ∩ B) + P (A ∩ B 0 )
1 1 1 1
0
P (A ∪ B) = + − =
2. P (B) = P (A ∩ B) + P (A ∩ B) 3 4 12 2

3. P (A0 ) = P (A0 ∩ B) + P (A0 ∩ B 0 )


4. Given that A and B are mutually exclusive events such
4. P (B 0 ) = P (A ∩ B 0 ) + P (A0 ∩ B 0 ) that P (A) = 0.5 and P (A ∪ B) = 0.9, find (i) P (Ā ∩ B̄)
and (ii) P (Ā ∪ B).
5. P (A) + P (A0 ) = 1
(i) P (Ā ∩ B̄) = P (A ∪ B)
6. P (B) + P (B 0 ) = 1

These relationships can easily be obtained by construct- P (A ∩ B) = 1 − P (A ∪ B) = 1 − 0.9 = 0.1


ing a contingency table.

Examples (ii) P (A ∪ B) = P (A) + P (B) − P (A ∩ B) Since A and


B are mutually exclusive, P (A ∩ B) = 0
19
1. Events A and B are such that P (A) = 30
, P (B) = 25 ,
and P (A ∪ B) = 54 . Find P (A ∩ B). P (A ∪ B) = 0.5 + P (B) − 0 = 0.5 + P (B)
From P (A ∪ B) = P (A) + P (B) − P (A ∩ B):
0.5 + P (B) = 0.9 =⇒ P (B) = 0.9 − 0.5 = 0.4
4 19 2
= + − P (A ∩ B) So, P (Ā ∪ B) = 0.5 + 0.4 = 0.9
5 30 5
19 2 4 31 4 7 Exercise
P (A ∩ B) = + − = − =
30 5 5 30 5 30 1: For events A and B, it is known that P (A) = P (B),
2. Events A and B are such that P (A) = 0.5, P (B) = P (A ∩ B) = 0.1, and P (A ∪ B) = 0.7. Find P (A).
0.7, and P (A ∩ B) = 0.3. Find (i) P (A ∪ B) and (ii)
P (A ∩ B̄). 2: The probability that a boy in S.5 class is in the football
team is 0.4, and the probability that he is in the volleyball
(i) P (A ∪ B) = P (A) + P (B) − P (A ∩ B) team is 0.5. If the probability that a boy in the class is in
both teams is 0.2, find the probability that a boy chosen
P (A ∪ B) = 0.5 + 0.7 − 0.3 = 0.9
at random is in the football or volleyball team.

3: Given that P (A) = 23 , P (B) = 21 , and P (A∩B) = 1


,
(ii) P (A ∩ B) = P (A) − P (A ∩ B̄) 12
find P (A ∪ B).
P (A ∩ B̄) = 0.5 − 0.3 = 0.2 4: If events A and B are such that they are independent
and P (A) = 0.3, P (B) = 0.5, find (a) P (A ∩ B), (b)
3. Events A and B are such that P (A) = 31 , P (A∩B) = P (A ∪ B). Are events A and B mutually exclusive?
1
12
, and they are independent events. Find (a) P (B) and
5: Events A and B are such that P (A) = 0.4 and
(b) P (A ∪ B).
P (B) = 0.25. If A and B are independent events, find
(a) Since A and B are independent events, P (A ∩ B) = (a) P (A ∩ B), (b) P (A ∪ B), (c) P (A ∩ B).
P (A) · P (B)
1 6: The probability of two independent events A and B
P (B) = occurring together is 81 . The probability that either or
4
both events occur is 58 . Find (i) P (A), (ii) P (B).

7: If two events A and B are independent and 3P (A ∪

Basics of Subsidiary Mathematics By Khalid M +256 754857296 khalidhkmsh@gmail.com 53


6. PROBABILITY THEORY

B) = 5P (B) = 4P (A), find (i) P (A), (ii) P (Ā ∩ B̄). are mutually exclusive. (a) P (John or Mark wins) =
P (John wins) + P (Mark wins) = 0.7. (b)
8: The events A, B, and C are mutually exclusive, such
P (neither John nor Paul wins) = P (John or Paul wins)C =
that P (A) = 0.3, P (B) = 0.2, and P (C) = 0.3. Find
1 − P (John or Paul wins) = 0.5.
(i) P (A ∪ C), (ii) P (A ∪ B ∩ C).
Example 3: A coin and a die are thrown together. Find
6.0.15 Probability Situations. These include:
the probability of obtaining: (a) a head (b) a number
The OR Situation: greater than 4 (c) a head and a number greater than 4
This situation deals with the probability of either one, or (d) a head or a number greater than 4.
the other, or both events occurring. If events A and B Solution: The sample space when a coin is thrown is
are two events, the probability that either A or B or both S = {H, T }, where H denotes ”head” and T denotes
occur is denoted by P (A ∪ B), where ”tail.”

(a) P (a head) = 21 .
P (A ∪ B) = P (A) + P (B) − P (A ∩ B).
The sample space when a die is thrown is S =
If the two events are mutually exclusive (meaning they {1, 2, 3, 4, 5, 6}.
cannot occur at the same time), the probability that A (b) P (a number greater than 4) = 13 .
or B will occur equals the sum of their probabilities, i.e.,
(c) P (a head and a number greater than 4) = 61 .
P (A ∪ B) = P (A) + P (B). In this context, the ”OR
8
Law” means probabilities are added. (d) P (a head or a number greater than 4) = 12
= 32 .

The AND Situation: Exercise

This situation deals with the probability of two events A 1. An ordinary die is thrown. Find the probability that
and B occurring together. In probability, the ”AND Law” the number obtained is: (a) even, (b) prime, (c) even
means that the probabilities are multiplied. For events A and prime, (d) even or prime,
and B, P (A ∩ B) = P (A) · P (B).
2. A and B are independent events, and P (A) = 0.3,
Examples P (B) = 0.75. Find the probability that: (a) Both A and
B occur, (b) A or B occurs,
Example 1: One ball is selected at random from a bag
containing 5 red balls, 2 yellow balls, and 4 white balls. 3. The probability that a student in a S.5 class is left-
Find the probability of selecting a red ball or a white ball. handed is 16 . From the class of 15 girls and 5 boys, a stu-
dent is chosen at random. Assuming that left-handedness
Solution: The two events are exclusive.
is independent of the sex of a student, find the probability
that a student chosen is a boy or is left-handed.
5 4. The probability of a student in S.5 getting an A in
P (red ball or white ball) = P (red) + P (white) = .
11 Subsidiary Mathematics is 0.24, and that of getting a B
in Economics is 0.28. What is the probability that a ran-
Example 2: John, Paul, and Mark compete in a 100m
domly selected student from this class will get an A or a
race. The probability that John wins is 0.3, Paul wins is
B?
0.2, and Mark wins is 0.4. Find the probability that: (a)
John or Mark wins (b) Neither John nor Paul wins. 5. A fair die is thrown twice. Find the probability of ob-
taining a 4 on the first throw and an odd number on the
Solution: Since only one person can win, the events
second throw.

Basics of Subsidiary Mathematics By Khalid M +256 754857296 khalidhkmsh@gmail.com 54


6. PROBABILITY THEORY

6. A bag contains 10 red balls, 5 blue balls, and 7 green P (A∩B) = P (A)·P (B). In this case, P (A/B) = P (A)
balls. Find the probability of selecting at random: (a) a and P (B/A) = P (B).
red ball, (b) a green ball,(c) a blue or a red ball,(d) a red
Example 1
or a green ball,
13 3
Given that P (H) = 52
and P (H ∩ G) = 52
, find
6.0.16 Conditional Probability. Conditional probabil-
P (G/H).
ity refers to the likelihood of an event occurring when we
have additional information about the occurrence of an-
other event. For instance, consider a school that employs P (G ∩ H)
P (G/H) =
both male and female teachers who teach both Arts and P (H)
3
Science subjects. If we randomly select a teacher and 52
= 13
know that the selected teacher is female, we might be 52
interested in the probability that she teaches a Science 3
=
subject. This scenario illustrates conditional probability. 13

In mathematical terms, if we have two events, A and B, Example 2


where the probability of A happening is not zero (P(A)
The probability of selecting a student in a certain high
6= 0) and the probability of B happening is also not zero
school who does Geography is 0.2. The probability of se-
(P(B) 6= 0), then we can denote the probability of A
lecting a student who does Subsidiary mathematics and
occurring given that B has already occurred as P(A/B).
Geography is 0.03. Find the probability of selecting a stu-
This can be read as ”the probability of A given B.”
dent who does Subsidiary mathematics given that he/she
Mathematically, it’s represented as: does Geography.

Let G be the event ”a student selected does Geography,”


P (A ∩ B)
P (A/B) = and let S be the event ”a student selected does Subsidiary
P (B)
mathematics.”
Similarly, we can calculate the probability of B happen-
ing given that A has already occurred as P(B/A), and it’s
given by:
P (G) = 0.2
P (S ∩ G) = 0.03
P (A ∩ B)
P (B/A) = P (S ∩ G)
P (A) P (S/G) =
P (G)
Since P (B ∩ A) = P (A ∩ B), these formulas can also be 0.03
=
written as: 0.2
= 0.15

P (A ∩ B) = P (A/B) · P (B) = P (B/A) · P (A) Example 3


1
If events A and B are such that P (B) = 12 and
It’s important to note that if events A and B are mutu- P (A ∩ B) = 61 , calculate (i) P (A), (ii) P (A/B), (iii)
ally exclusive (i.e., they cannot happen simultaneously), P (A/B).
then in such cases, P (A/B) = 0 and P (B/A) = 0. On
the other hand, if events A and B are independent, then

Basics of Subsidiary Mathematics By Khalid M +256 754857296 khalidhkmsh@gmail.com 55


6. PROBABILITY THEORY

ber” and M be the event ”the card picked shows a mul-


1 tiple of 4.”
P (A ∩ B) 6 1
(i) P (A) = = 1 =
P (B) 12
2
1
P (A ∩ B) 6 1
(ii) P (A/B) = = =
P (B) 1
2 E = {2, 4, 6, 8, 10, 12, 14, 16, 18, 20}
3
1
P (A ∩ B) 6 1 M = {4, 8, 12, 16, 20}
(iii) P (B/A) = = 1 =
P (A) 2
3 |E| 10 1
P (E) = = =
|S| 20 2
Example 4 |M | 5 1
P (M ) = = =
|S| 20 4
A card is picked at random from a pack of 20 cards num- |M ∩ E| 5 1
bered 1, 2, 3, . . . , 20. Given that the card shows an even P (M ∩ E) = = =
|S| 20 4
number, find the probability that it is a multiple of 4. P (M ∩ E) 1
1
P (M/E) = = 41 =
Let E be the event ”the card picked shows an even num- P (E) 2
2

CONTACT FOR COMPLETE BOOK

Basics of Subsidiary Mathematics By Khalid M +256 754857296 khalidhkmsh@gmail.com 56


ABOUT THE BOOK
"Basics of Subsidiary Mathematics" is a carefully crafted
textbook tailored to meet the educational needs of A Level
Submaths students. Aligned with the 2020 NCDC syllabus, it
serves as an invaluable tool for both educators and learners
alike. With its clear explanations, illustrative examples, and a
wealth of exercises, this textbook is designed to instill a
profound grasp of fundamental mathematical concepts.

The book seeks to empower students. It equips them not only


with the knowledge required to excel in their mathematical
studies but also with the problem-solving skills essential for
success in various educational pursuits and real-world
applications.

You might also like